Open Thread 76.25

This is the twice-weekly hidden open thread. Post about anything you want, ask random questions, whatever. You can also talk at the SSC subreddit, the SSC Discord server, or the Cafe Chesscourt forum.

This entry was posted in Uncategorized and tagged . Bookmark the permalink.

321 Responses to Open Thread 76.25

  1. WashedOut says:

    Why do gay men affect a feminine persona and lesbians a masculine one?

    • Jiro says:

      Costly signalling. (Costly with respect to being perceived as normal by outsiders.)

    • dndnrsn says:

      Do gay men affect a feminine persona? By many accounts, masculine men, and “straight-acting” men are considered highly desirable on the gay dating market, and you will see men (on, for example, Grindr and similar apps) state that they are not interested in effeminate men.

      • Jiro says:

        If gays are not accepted, there isn’t much of a gap between being out as a weirdo and out as a normal person. Both actions are costly. If gays are accepted, there’s a much bigger gap. In other words, the relative cost of the signal goes up the more that gays are accepted.

  2. the verbiage ecstatic says:

    If anyone was wondering what this was about, you can now see for yourself by going here: https://sscnotify.bakkot.com/subscribe?author_name=INSERT YOUR NAME HERE

    • CatCube says:

      I signed myself up for replies to my comments, but I don’t know if I’ll be able to give too many reports on how well it’s working, since I’m at a place where the internet sucks. (MyFi that goes in and out every couple of minutes)

      Thanks for doing this. It’s always been tough following threads you’ve been a part of and this should make it a lot easier.

      • the verbiage ecstatic says:

        Well, let’s see if you get this reply 🙂

        Even on a fast connection, replies are going to be a little slow.. there seem to be some limits on how frequently I can pull the latest comments from the WordPress API. So this is less for real-time updates and more for knowing if someone replied to you after you logged off.

        • CatCube says:

          It worked. I got the reply around 2338 EDT.

          The only thing I can see, and I’m basing this on my understanding of the instruction page for signup, is that a 2nd-deepest level post is going to get hammered with e-mails when a long series of threads develops in the deepest level, where most of them are not technically replies to the 2nd deepest level post.

          Maybe in the deepest level, don’t send the e-mail to the “Reply” post, but only to the people called out with @Username? The issue there is that we’re not consistent with using the @Username for replies here.

          • Paul Zrimsek says:

            The time may have arrived for an improvement several of us have suggested in the past: give deepest-level comments their own Reply button, which instead of further indenting would append the new message at the same (deepest) level, and prefill it with a “@Username [time]” line.

          • CatCube says:

            I don’t know how difficult it would be, since it’s possible to reply to a comment at the deepest level by copying the URL from the “Reply” button of another post and hand jamming in the comment ID of the deepest-level comment (as I’ve done here). So I wonder if the no reply to the deepest level is implemented by simply turning off the reply button on the deepest level, which could be turned back on.

            I’m doing it partly because I’m curious to see what will happen with the e-mail reply app.

          • random832 says:

            How about automatically quoting whatever text is selected, as well?

            @CatCube – the issue (and ISTR a few people complained about someone doing it, a couple weeks ago) is that doing this violates people’s expectations that replies at the deepest level will show up in chronological order.

            Maybe what needs to happen is to have the code take care of this as well, and then the reply button can be enabled at all levels.

          • CatCube says:

            @random832

            I’d push back on that one. My browser already makes my computer scream like a jet engine loading all the client-side code for the comment section, so adding another feature that’s easily duplicated isn’t something I’m eager for.

            The collapsing already-read comment threads and the e-mail reply here are two that I can’t easily duplicate myself, so I’d rather have my computer tied up doing those things.

          • the verbiage ecstatic says:

            My thought is let’s err on the side of spamming the person who writes the deepest reply-able post, since that person is *probably* interested in the thread, and then if people start getting better about using @-replies consistently (which they might once there’s real consequences to using / not using them), we can change things up.

          • random832 says:

            I suspect you’re ignoring that there’s a huge difference between the features like highlighting, checking for ones you can edit, etc, etc that it has to iterate through all the comments actually running something, vs something that only runs once only when you press the reply button, and assuming the latter (loading a piece of code that sits idle until you press a button and then only looks at the selection) would contribute noticeably to CPU usage because the former (which is actually running something, hundreds of times, on a long comment thread) does.

            As it is, it’s very annoying to have to go back and forth (the reply box is often distant from the comment I am replying to) to copy multiple things (at least the author’s name and quoted text), and I don’t believe your assumption that it would add a significant amount to load times.

            P.S. This whole time I thought you were talking about the “automatically quote selected text” feature I suggested. It just occurred to me that you might have been responding to my “have the code take care of this as well” w.r.t. reordering posts so that posts in the flat section appear in chronological order – that suggestion was actually to have the server-side code do this. (And server-side code will perform much better anyway because it isn’t bogged down by having to recalculate the whole page layout whenever it changes the size of something)

  3. Machina ex Deus says:

    Feminist philosophy journal Hypatia calls for retraction of paper it published. Or does it? Chronicle of Higher Education

    Note: I had to get up halfway through reading it to make popcorn.

    • Zorgon says:

      Dominant alliances have such a highly predictable pattern, don’t they?

    • Zorgon says:

      That said, the last paragraph leads us to the possibility of a Shining City on a Hill where the word “violence” is no longer so relentlessly abused.

    • Machina ex Deus says:

      Sorry, I should have included an excerpt; it starts:

      The associate editors of Hypatia, a journal of feminist philosophy, apologized on Facebook this week for the publication of “In Defense of Transracialism,” an article by Rebecca Tuvel, that had quickly drawn opprobrium. “Clearly,” they wrote, “the article should not have been published.”

      One problem with that statement: Hypatia’s editor now says she disagrees with it.

      My favorite part is basically all of it.

    • Aapje says:

      It’s pretty amazing to see people rationalize their witch hunt:

      Claire M. Colebrook, a professor of English, philosophy, and women’s, gender, and sexuality studies at Pennsylvania State University, said that if she had read the article outside the context of the controversy, she would have thought it was simply “overly abstract.”

      Now, though, she agreed with her colleagues’ arguments that the article implied that black philosophers’ work does not matter.

      It’s like chickens, once a fellow chicken is wounded, they can’t help but pick at the wound. It’s an instinct.

    • CatCube says:

      Wow. So apparently using the given name of a transsexual has had the term “deadnaming” attached to it. This trans shit is getting more Orwellian by the day.

      • Aapje says:

        There is some sense to it, as using the ‘old’ name can ‘out’ a trans person. However, there is a strong tendency not to think each case through, but apply the rule indiscriminately. So you get this nonsense where Jenner gets treated as if there is a single person alive who doesn’t know.

  4. the verbiage ecstatic says:

    I’m testing the comment reply notification feature we’re building. Am going to post a few replies to this…

  5. caryatis says:

    Does anyone have opinions about laser eye surgery? I’m considering getting it but I’m concerned about reports of long-lasting eye pain afterwards. Also, the FDA says “It is also important to note that no laser company has presented enough evidence for the FDA to make conclusions about the safety or effectiveness of enhancement surgery.”

    • hoghoghoghoghog says:

      I am also wondering about this. I would absolutely pay $5,000 for this if it had less than a 99.9% chance of screwing up tear-production or permanent halos or whatever. But when I’ve looked up information on this everyone seemed extremely squirrely.

    • I had Lazik surgery I think 14 years ago. Worked great for me. I’ve needed glasses since I was in third grade, and had a rather large correction before my surgery. Now I don’t need glasses at all. Well I use cheaters for small print and I need glasses for driving at night so I can see signs. But I am 60 years old so I think Lazik didn’t cause those issues.

      I didn’t get Lazik until it appeared to me that it was no longer experimental and very rarely had issues. I did not use the discount Lazik places because I figured that they would have a higher incidence of problems. I don’t know whether or not I was justified in that.

      Obviously my experience is totally anecdotal, so if you are worried about the risk of bad things happening, this won’t tell you anything useful about that. But I can tell you there is a high upside.

    • cassander says:

      I just got it a few months ago. My dad had it done about 10 years ago by a surgeon who fixes the screw ups of other surgeons. He recommended the guy I went to, and I have to say I’m very happy with it. Vision is great, pain was largely gone after a couple days, and that halo-ing seems to be fading, though slowly. They also give you a guarantee that if you ever need an adjustment, they’ll do it for free, which is something I found very reassuring. I can’t speak for the cheaper places, but if you’re willing to shell out a few grand for it, I’d say go for it.

    • psmith says:

      Cosigning for “interested, but wary.” For what it’s worth, I’ve never known anyone IRL first- or second-hand who had problems.

      PRK seems more robust than LASIK, albeit with a longer healing time and usually a slightly higher price.

      Implantable contact lenses also seem like they might be a better technology and I think they can correct astigmatism now, but you’re looking at more like $10k for that.

    • Brad says:

      I’ve have four immediate family members that got eye surgery. Three are completely satisfied, one gets halos at night and dry eyes. Even that one is not upset he got the surgery.

    • keranih says:

      Seriously looked into it about fifteen years ago, but decided against for cost. Looked into it a LOT more seriously about eight years ago, but couldn’t make a job conflict work out and had to pass.

      Since then I’ve had to shift to bifocals, which turned out to be far less stressful than I had thought. Looking back on my life, I’ve spent a lifetime active and outdoors, and the glasses have been the least of my issues.

      I know multiple coworkers and family members who have had it. No significant complications, but the healing process seems to drag on for a lot longer than the clinics advertise. My mom had night driving issues for more than a year.

    • Eric Rall says:

      I had LASIK about four years ago, and my wife had LASIK two years ago. We both had very good vision results, about the same as my best corrected vision pre-surgery (no problem reading 20/20 line and can strain to read most of the 20/15 line for me, and 20/20 with slight uncorrectable (higher-order aberration) astigmatism for her). We both have reduced tear production, which has tapered off for both of us, a bit faster for her. I need eyedrops once or twice a day, and she needs them rarely when the air is dry or her allergies are acting up. Despite this, we’re both very happy with the overall results, as we find occasional eyedrops much less annoying than dependence on glasses and contacts.

      Things to look into when choosing a surgeon and a surgery approach:

      1. Surgeon’s reputation and experience. From what I gather (my own research and talking to surgeons when choosing one), LASIK and PRK are very safe in terms of severe complications (lasting damage to your best corrected vision or dryness/pain complications that can’t be managed effectively through eyedrops and tear duct plugs) if you’re a good candidate for the procedure, you have the procedure done by an experienced and reputable surgeon, and you follow the aftercare instructions. A key piece of the aftercare instructions that a good surgeon should emphasize is to rest with your eyes closed (sleeping if possible) most of the rest of the day after surgery: multiple studies have shown that bad outcomes are much, much less likely if you do this. The instructions about cleaning your eyelids with antiseptic wipes before surgery, using antibiotic and anti-inflammatory drops after surgery, and keeping out of water a certain length of time after surgery are important as well.

      2. LASIK vs PRK. LASIK cuts a flap into the surface of the cornea and performs the vision correction part of the surgery under the flap. PRK does the same vision correction (same technology and same vision quality results) directly on the surface of the eye. The main difference is that LASIK requires a thicker cornea to perform safely (I think something like 70-80% of people have thick enough corneas for LASIK), and LASIK severs nerves within the cornea (this is probably most of what interferes with tear production: the nerve heal, but slowly and somewhat imperfectly) but leaves the protective outer layer of the eye (the epithelium) intact. PRK burns right through the epithelium, but leaves the corneal nerve unsevered. The epithelium grows back, but takes about a week to do so. The net result is that LASIK has more dry eye symptoms (this isn’t 100% confirmed, but the theory points to it, and there are suggestive but not definitive clinical results pointing to this), but PRK has much longer short-term recovery (your vision is close to your previous best-corrected vision in 1-2 weeks with PRK instead of 1-2 days with LASIK). Most surgeons seem to recommend LASIK unless you have bad dry eye symptoms pre-surgery (the drier your eyes pre-surgery, the more at risk you are for dryness-related complications post-surgery) or unless your corneas are too thin for LASIK.

      3. If you’re a candidate for LASIK and opt for it over PRK, there are two schools of thought about orientation of the flap: superior (the flaps swing up towards your forehead) and nasal (the flaps swing in towards the bridge of your nose). Superior flaps are significantly safer in terms of flap complications (the flap gets knocked out of place or (worst case) torn off before it heals back into place), but nasal flaps have less incidence of dry eye complications. The theory is that the act of blinking will push superior hinges back into place if they’re a little out of place (they heal back in place in a matter of days, but there’s a window of vulnerability), while blinking perturbs nasal hinges; however, nasal hings preserve some of the key nerves that help regulate tear production.

      4. There’s a big difference between “Wavefront Optimized” and “Wavefront Guided”: Optimized uses a standard pattern to avoid inducing the higher-order aberrations (refractive errors other than far- or near-sightedness or regular “first-order” astigmatism) that older forms of LASIK/PRK induced (this was the main thing that caused permanent halos — both types of Wavefront LASIK cause temporary halos due to inflammation during the healing process, but this goes away completely fairly quickly), while Guided also attempts to customize the pattern to also correct for some existing aberrations. Guided is significantly more expensive, but (as of two years ago) it’s an open research question to what extent it produces better results (the prevailing opinion I found at the time was “probably better, but the differences are subtle”, with minority views for “more than a bit better” or for “pretty much the same”). I opted for Guided, but do your own research and make your own decision if you decide to go for surgery.

    • Reasoner says:

      My friend got lasik maybe 5-10 years ago and periodically found himself contemplating suicide because of how it screwed with his vision. Then again, this wasn’t the only reason he found himself contemplating suicide. Also the tech might be better now.

      Does anyone have opinions on this by the way? http://gettingstronger.org/2010/07/improve-eyesight-and-throw-away-your-glasses/

  6. Anonnymous says:

    I’m writing a blog and was wondering if my text should be justified or not. I did some digging back in the day that led me to use justified, but today I noticed that our esteemed host uses the default left align and it has never bothered me. I have on the other hand noticed when justified text leads to weird spacing between words on some sites. So on the principle of using what seems to work, I should probably ditch justified.

    • MNH says:

      I think justified is horrendous. In particular, it makes rivers unbearable when they occur. There are methods for dealing with this if you have fine control over your spacing and don’t mind spending some time, but I can’t imagine this ever being worth the work.

    • random832 says:

      I have noticed that justification works much better when hyphenation is used. More browsers support auto-hyphenation than did, say, five years ago, but it’s still not guaranteed (you may need to set the page language).

  7. Tibor says:

    Apparently, the French have also voted their version of Donald Trump to the presidential office. Except that he is wrapped up in a young hip pseudo-liberal package and marketing is sadly everything in politics. Somehow, I find it even more annoying than Trump, who at least does not pretend to be something else than what he is. Macron’s level of chutzpah are off the charts.

    • Dabbler says:

      I thought Le Pen was the French Donald Trump and Macron was the French Bernie Sanders?

      • HeelBearCub says:

        Macron is definitely not Bernie. He is the technocratic, middle-of-the-road, started-my-own-party guy.

        More Bill Clinton than Sanders. Mélenchon is perhaps the Sanders-ish guy you were thinking of. He is a a socialist and did not endorse Macron in the run-off.

        • Tibor says:

          Méchelon is a communist, way more left even than Sanders (or pretty much anybody in the US outside of academia 🙂 ).

          Sanders did essentially endorse Trump-like immigration politics though IIRC, he wants a extensive welfare state but he said that he wants welfare first and foremost for Americans. In that he is somewhat similar to Denmark.

          I’m not entirely sure what Macron is about. His most salient characteristic seems to be promoting his own career. He said something about liberalizing the (extremely overregulated) French labour law (a tall order in a country controlled so firmly by unions), but now he seems to want to solve high French unemployment by removing foreign competition instead.

          • hoghoghoghoghog says:

            Sanders did essentially endorse Trump-like immigration politics though IIRC, he wants a extensive welfare state but he said that he wants welfare first and foremost for Americans. In that he is somewhat similar to Denmark.

            That was my impression too, but I just looked it up and his stated opinion on immigration is reasonably liberal by the US’s current benighted standards: https://berniesanders.com/issues/a-fair-and-humane-immigration-policy/

          • Tibor says:

            For some reason I keep confusing Mélanchon and Michelin and write (and say) Méchelon…

    • HeelBearCub says:

      Macron is talking about legal differences in wages though.

      [Eastern European workers] can be posted to France for limited periods and paid at eastern European wage levels which are often below guaranteed minimums in the west.

      Subversion of minimum wage laws is often criticized from the left, and isn’t motivated by xenophobia.

      • Dabbler says:

        Incidentally, other than the obvious Duterte which world leaders or candidates for office do you think deserve to be considered their nation’s equivalent to Donald Trump?

        • ChetC3 says:

          Not HBC, and he’s been out of office for a while, but Silvio Berlusconi seems quite Trump-esque to me. Does anyone know if Diego Maradona is considering a political career? He could make a pretty good Trump analogue if he had a mind to.

          • Tibor says:

            Yeah, or more accurately – Trump is an American Berlusconi (Berlusconi was first, come on 🙂 ). Before his elections people made ridiculous comparisons of him to Musollini or Hitler but comparing him to Berlusconi is actually quite accurate…except that Trump does not have sex with underage girls as far as we know 🙂

          • Matt M says:

            except that Trump does not have sex with underage girls

            This is KIND OF an important distinction, is it not?

            It’s like saying “this guy is like Hitler except he doesn’t kill Jews.”

          • Loquat says:

            It’s like saying “this guy is like Hitler except he doesn’t kill Jews.”

            Was having sex with underage girls the primary problem with Berlusconi? I admit I haven’t paid much attention to Italian news, but when he was in office I remember hearing a fair bit about financial conflicts of interest, passage of laws intended to benefit himself, open quarrels with the judiciary, ties to organized crime, and sex scandals involving adult women, all of which are also pretty bad, and some of which Trump definitely has going on himself.

          • Matt M says:

            As an American, it’s the only thing *I* recall hearing reported about him. Perhaps in Europe the perspective is different.

          • Aapje says:

            I’m sure the Germans didn’t like it when he called on of their politicians a concentration camp guard (kapo).

            He is also known for being charged in dozens of legal cases and managing to not get convicted somehow on most of them.

          • herbert herberson says:

            “except that Trump does not have sex with underage girls as far as we know”

            He was known to take the odd plane flight on Jeffrey Epstein’s Lolita Express.

            (As was Bill Clinton, for fairness’s sake)

          • Tibor says:

            @Matt M: Not really, this was really just the last straw. Berlusconi politically survived countless scandals and legally dubious things before that. But while the Italians might be a bit more tolerant to escapades like that than others, it was just piling up.

            What makes Berlusconi similar to Trump though is that he is also a media magnate, he seems to be similarly vain and he also presented himself as a sort of a “people’s voice” even though he is a very rich man.

          • Whatever Happened To Anonymous says:

            Does anyone know if Diego Maradona is considering a political career? He could make a pretty good Trump analogue if he had a mind to.

            Plz no, he already destroyed our window for winning a WC this generation, no need to have him fuck up the country.

          • Matt M says:

            he seems to be similarly vain and he also presented himself as a sort of a “people’s voice” even though he is a very rich man.

            This description probably applies to like 80% of politicians ever. Including beloved ones such as Abraham Lincoln and JFK.

          • rlms says:

            @Matt M
            I don’t think it applies to Lincoln, see here. And if the wealth column is accurate, it applies more to Trump than others.

      • Tibor says:

        I think it is motivated more by protectionism than xenophobia. The target is clearly preventing foreign competition and the chutzpah is that it is wrapped in “helping the poor” rhetoric.

        Aside from that, what falls into this category is a Polish truck driver who delivers something to France – he is employed in Poland and his job is regulated by Polish laws – including his wage.

        And the second article I linked to is another form of protectionism – this “buy European” nonsense is about preventing non-EU companies in competing for public projects in the EU. It seems to be at least partly a “retaliation” for Brexit.

        Both of these policies are nothing but protectionism but while Trump gets (rightly) criticized for that, Macron is a “convinced European” (as the eurofederalists like to call themselves) and does token “progressive” things like having 50% of the ministers female, hence gets a free ride.

        • Matt M says:

          I think it is motivated more by protectionism than xenophobia.

          It’s called “protectionism” when the left does it and “xenophobia” when the right does it, but it’s all the same thing…

          • hoghoghoghoghog says:

            People absolutely focus too much on movements’ motivations rather than their actual actions. That said, there is at least a real difference in motivations here. If the left were assured that immigration would not hurt the employment prospects of current residents, they would all accept far open-er borders. The right would not. It’s wrong to call that xenophobia insofar as the right has other reasonable concerns. But there is a real difference between the goals of the right and the goals of the left.

          • Matt M says:

            If the left were assured that immigration would not hurt the employment prospects of current residents, they would all accept far open-er borders. The right would not.

            I’m not so sure about that. Isn’t “dey took our jerbs” a typically mockery of the right?

            Both sides are primarily motivated by economics imho, it’s just that the left prefers economic meddling (tariffs, minimum wage laws, licensing) that hurt foreign labor in less overt and obvious ways, while the right prefers, you know, a giant concrete wall.

          • Tibor says:

            I’m not sure. I think people call Trump or Le Pen protectionist. I think it is called protectionism when people want to emphasize economics and xenophobia when they want to say that the people don’t like the foreigners for other reasons. With Macron’s “Buy European act”, calling it xenophobia would be ridiculous even if it was Trump’s plan (well, “buy American” is what he says after all).

          • Matt M says:

            xenophobia when they want to say that the people don’t like the foreigners for other reasons

            With the exception of “we are concerned that some of them might be terrorists”, what “other reasons” does the right ever actually cite?

            I feel like “you just hate foreigners because they’re different” is a left-wing smear of the right… not an accurate summary of their position.

          • Tibor says:

            @Matt M: Perhaps, but I was referring to what people say about other people. And sometimes the conservatives actually are overestimating the dangers of terrorism or “islamisation” etc (especially since giving in to the demands of radical islamic groups in order to be “racially sensitive” is not necessarily a part of allowing more foreigners from muslim countries, it is more of a problem of over the top political correctness, which is a domestic issue, rather than those foreigners themselves).

            And I am not saying there are not real issues with immigration, particularly when it is an immigration from a very different and in many ways backwards culture and combined with a generous welfare state. Nevertheless, there are conservatives whose reaction is out of proportion and most of the left will know mostly about these, because the media you read give you what you want to read and most people prefer to read about how bad the other tribe is rather than having to think about details and nuances.

          • Conrad Honcho says:

            @Tibor

            Whether fear of islamification is an overreaction or not depends on how bad the potential downsides of islamification are. Conservatives have listened to Brigitte Gabriel talk about the civil war of Islamic aggression in Lebanon. When the Islamists had the numbers, they came to where the Christians were, and the liberals said “but we were your friends, we welcomed you, we defended you!” and the Muslims said “you were very stupid!” and slaughtered them. Ripped the limbs off of infants even.

            So we look at the potential upsides of Islamic immigration and see: kebab stands maybe? And the potential downsides: dismemberment of children, civil war and complete collapse of society. And say “this does not seem worth it.” I mean, I’ve been to Egypt. I love me some shawarma. Not that much, though.

          • rlms says:

            @Conrad Honcho
            Please quantify “islamification”. What proportion of Muslims are you talking about?

          • Tibor says:

            @Conrad Honcho: The sectarian violence in Lebanon was way more complicated than “good peaceful Christians massacred by evil violent Muslims”. Lebanon is perhaps one of the most religiously diverse countries in the world and there were violent militias on all sides. The militias would stop civilian buses and shoot passengers who adhered to the “wrong” religions (wrong depends on the militia). Conveniently, religious adherence was listed in the national ID card back then (now it is illegal to do that there). I think it was actually the Christian militias who started this particular practice, although it was preceded by other horrible atrocities form both Muslims and Christians.

            Is by and large Islam more dangerous today worldwide than Christianity? Yes. Does that mean that say Germany is in danger of becoming a Muslim theocracy (or at least a country heavily influenced by Islam). Not likely. Yes, sometimes there is a really idiotic story, typically of a Green Party politician in office giving in in a very badly understood attempt at tolerance to an incredibly stupid demand by some Islamic groups who feel offended by various things they have no business to complain about, but these make it to the headlines exactly because of their ridiculousness. And like I said, a whole different question is whether it is a good idea to take in a lot of uneducated immigrants with no job prospects to live off your welfare state, especially if they come from a rather incompatible culture and this is the best way to make sure they don’t assimilate (and also you might say that actively promoting differences and almost discouraging assimilation is a very bad policy), but this is still different than worrying about Germany turning into Syria. I mean some people really give me the impression that they believe that all the Muslims are going to slit their throats the moment they lose guard, in which case the “-phobia” is actually sort of justified.

  8. ckrf says:

    Is anyone in Myanmar? I’m in Yangon for the summer–would love to see what kind of SSC readers are to be found around here.

  9. James Miller says:

    My Netflix account was hacked and someone unauthorized used it to frequently watch Star Trek Enterprise episodes. I changed the password on that account and on the email account linked to the Netflix account. Should I assume that this email account has probably been hacked as well? Should I contact my bank and be especially worried about identity theft?

    • MNH says:

      If your email is like mine (gmail) I believe you should be able to see a log of log-ins and their time and maybe location. Perhaps that could help you determine this.

      • Aapje says:

        I once got warning when a person from the other side of the world accessed my gmail account. Their hack-detection seems quite decent.

        • Fahundo says:

          I’ve been to the other side of the world and used my gmail without getting a warning. How do they know?

          • rlms says:

            Probably from if the device you’re using changes.

          • The Nybbler says:

            I’ve been to the other side of the world and used my gmail without getting a warning. How do they know?

            Perhaps they saw the receipt for your plane ticket.

    • Matt M says:

      This happened to me once and nothing major came of it. You should assume the particular combination of e-mail and password you used on Netflix has been compromised, and will be attempted to be used at any wide variety of websites, so anywhere else that uses the same e-mail/pass will be compromised as well. If you use a unique password for everything you’re probably fine.

  10. Mark says:

    I know it’s highly likely this is complete bullshit, but I’m getting a raging knowledge-horn from my own comment on the other thread:

    Focus can travel faster than the speed of light – like, if I have a powerful beam of light and wiggle it around, the end of the beam is travelling faster than the speed of light. I can look between two stars faster than a message might be sent between them.

    So, couldn’t I use this to “travel” faster than the speed of light, by bringing together whatever information I receive from both stars at my third point faster than the two points could communicate with each other?

    If we’re concerned with the person making the observation rather than where they are doing it, you could half the time of any information transfer by shooting them off towards the information they want. If we can recreate any location informationally does that mean we can travel FTL?

    It is a matter of how much information we can have about a place.

    If we could have all the information about a location, we could send that information and (effectively) travel FTL.

    If we don’t care about much of the information, we can definitely travel FTL. And, in fact, this solves the Fermi paradox. Why isn’t the universe teeming with life? Because we are the result of too many FTL message transfers where we could only pay attention to the most local of phenomena – all of the rest was just assumed to be boring.

    I’ve had a few beers, but I feel as if this is the most genius comment ever. Please comment upon these findings to either prevent unnecessary excitement, or to celebrate my discoveries.

    • Incurian says:

      Are you sure it was just beer?

      • Mark says:

        Dude… don’t do this to me. From my perspective, I’ve legit solved the Fermi paradox and/or found a way to travel faster than light.

        I need to know why this is wrong.

      • Incurian says:

        While I concede that my intelligence and character are lacking, the answer is 3 and I didn’t want to deprive you of that moment of clarity. If you sober up and would still like an explanation, we can talk.

      • Incurian says:

        Also, this is evidence you might not be thinking clearly at the moment.

    • smocc says:

      Do some calculations and I think you will quickly see what’s wrong. You don’t even need relativistic effects, just that light travels at a fixed speed.

      If I understand you, this is what you’re saying. A wants to send a message to B, but they are separated by a large distance, D_AB. A decides to save time by using an intermediary beacon, X, at a distance D_AX from A and D_BX from B. When A wants to send a message, they first send a pulse of light to X. Then X rotates a laser that was pointing at A so that it points at B. When B sees the laser from X, they know a message has arrived.

      How long does this take? First X has to wait to see the pulse from A. That’s D_AX/c. Then X takes a little time to rotate its laser, call that amount of time T.

      Now when does B see the laser from X arrive? For B to see the laser, the light still has to travel from X to B. So the time B sees the message arrive is D_AX/c + T + D_BX/c.

      This is always longer than D_AB/c, the time it would have taken to send a message at light speed directly to B.

      The key is that B cannot receive information unless light has traveled from a source to it, and that always takes time. A laser dot appearing to move faster than the speed of light is entirely illusory.

      • Mark says:

        No…

        I feel like you’ve overcomplicated that a little.

        So, I’m not saying that sending a message to A -> B -> C is faster than sending a message A -> C.
        I’m saying sending a message A ->B<-C is faster.

        That is, if you have three points on a line A—B—C and you can send all of the information that makes A what it is, to B, and you really wanted some information from C, you would send A to B and get your message in half the time.
        I am at A. I want to know something that is at C. I travel to B to get the message faster.
        I communicate all of the important information about A to B, I've "travelled" faster than light with respect to the information from C.

        And you could then send a message back to C in less time as well.

        • smocc says:

          Sending a message A-> B <- C is faster than what? What are you comparing to?

        • Loquat says:

          That sounds like it really only works in the extremely specific circumstance that nobody at A needs to know the requested information from C apart from whoever will be travelling to B.

          • smocc says:

            And even then it doesn’t allow C to know when A wants the information. If C waits until A decides it wants the information a message needs to be sent A->C somehow. The other option is that C sends the information in advance, but then you’re not really sending a message C->A, but B->A, which of course is faster.

          • Mark says:

            Yeah, I think I’m going to have to rethink this. It’s a possible explanation for the Fermi paradox, though.
            If there were some person who were interested in getting some information from some specific place maybe he’d send himself (informationally) to some other point where he could be instantiated to get that info quicker, but then, why not just generate the info he wanted, why would he want it, etc.

            I don’t think what I’m saying is wrong, it’s just hard to think of a motivation to do it.

          • FacelessCraven says:

            my understanding is that, rigorously, the sort of “travel” you are describing is possible, but utterly useless. Any possible use by definition requires transferring information, which cannot be done faster than light-speed. you can move the laser’s point of impact faster than light, but the laser only moves at light-speed, so those changes only propagate at light-speed. You can get to B faster than C, but you can’t get a message to C faster, so they won’t know to send the message to B until the light-speed message catches up, and so on.

    • Matt M says:

      I don’t know anything about science to judge the merits of your theory.

      But I choose to believe that if and when FTL travel is solved, it will be made public first via an SSC comment thread.

    • Loquat says:

      If we don’t care about much of the information, we can definitely travel FTL. And, in fact, this solves the Fermi paradox. Why isn’t the universe teeming with life? Because we are the result of too many FTL message transfers where we could only pay attention to the most local of phenomena – all of the rest was just assumed to be boring.

      Practicality of your proposed travel method aside, I have no idea what the hell this last paragraph even means. Can you explain it further, possibly tomorrow after sobering up?

      • Mark says:

        If you needed me to make an important decision about some information coming from a distant star, you could send me somewhat in the direction of that star to get the information sooner.

        If the structure of my brain can be observed and described, you could send me as information to an existing base somewhere half-way between my origin and the information source, and have me instantiated.

        Likewise, at this halfway point, you could use the information contained within my mind (or other information) to instantiate a low information fidelity version of this world, and then instantiate a visitor mind (from a distant star) within this world. So the visitor mind could travel faster than expected to a world.

        Hmmm… is this like time dilation?

        Anyway… since part of the information that makes a world what it is is its position with respect to all the other stars, I think that no matter how detailed the simulation, you would necessarily lose some information with respect to other distant stars.
        The less detail you need in your simulated world, the easier it would be to achieve.
        So maybe we’ve been sent us off to some point in the galaxy to receive an important message, or visitor, but, in our simulated world, all of the other stars have been turned into bog standard no-life worlds, to cut down on the information sent, or because it isn’t possible to correctly incorporate information from other stars to our original position within our simulation in the half-way point world.

        But, I guess once you start getting into simulated worlds literally anything goes and you can say anything you like. Classic pub chat.

        • Loquat says:

          Merely sending information to an uninhabited rock does nothing. You need a pre-existing base with appropriate technology, maintenance crew, etc, to do any “instantiating”. So, if Earth is the result of such a process, where’s the tech that made us? And why did our senders include a whole planet full of people that have no idea any of this is going on?

    • Reasoner says:

      Focus can travel faster than the speed of light – like, if I have a powerful beam of light and wiggle it around, the end of the beam is travelling faster than the speed of light.

      This sounds incorrect to me.

      Let’s say I have a garden hose and I’m watering my garden. Initially I’m pointing the hose at Plant A and the water is spraying Plant A. Then I quickly spin around to spray Plant B behind me. There’s going to be some lag as the destination that the water sprays catches up to the new direction that my hose is pointing. You can see this lag if you observe the trajectory of the stream: as I am spinning, the trajectory will have a curve to it instead of being straight.

      This is true for light too, but it’s not relevant to your everyday practical experience of light because light travels so damn fast.

      I’m not a physicist, but I think the travel time for the end of the beam will equal the sum of the time it takes for you to rotate the beam and the time it takes the particles of light to travel from you to their new destination. For simplicity let’s assume the rotation of the beam is instantaneous. Let’s say you’re at Star B and you want to receive some info from Star A and pass it on to Star C. You will have to wait for Star A to transmit you the info before transmitting it to Star C. So what you’re doing is functionally equivalent to holding up a giant mirror and reflecting the message you get from Star A to Star C. Due to triangle inequality, the total distance the light travels will always be at least as large as it would have to travel to go from Star A to Star C directly.

      Did that help?

      • random832 says:

        I’m not a physicist, but I think the travel time for the end of the beam will equal the sum of the time it takes for you to rotate the beam and the time it takes the particles of light to travel from you to their new destination.

        No. It’s only the time it takes for you to rotate the beam. The (much longer) time it takes the particles of light to travel will pass before the end of the beam starts moving.

        To get “the speed of the end of the beam” (or water stream) you measure the time from when the last light (or water) hits the original destination to when it first hits the new destination, not the time from when you start moving. No physical object (wavelike or otherwise) is traveling faster than light, but the “concept of the end of the beam” – which is not composed of the same photons from one moment to the next – is.

    • rlms says:

      Think about an adjustment of this plan, where instead of pointing between two stars, you use pointing at one star vs not pointing at it to send data faster than the speed of light (allegedly).

      • smocc says:

        Still doesn’t work. Imagine A sends a bit of information by either turning on or turning off a light pointed at B. This is equivalent to pointing the light at B or not.

        A decides to send a message to B, and so they turn on their light (or point it at B) at t = 0. When does B see the first photon that was aimed at them? At t= D_AB/c

        Alternatively, A is already sending a steady stream of photons to B. They send a message by turning the light off at t=0. B only knows they’ve received a message after the last photon to leave A arrives at B. When does that happen? At t = D_AB/c

        • rlms says:

          Sorry, I should’ve been clearer. I meant “think about” in the sense of “think about how an adjustment of this plan leads to an obvious contradiction (sending information from A to B faster than the speed of light) in order to improve your understanding of your original example”, not “think about this cool adjustment to your plan that will let you send information from A to B faster than the speed of light”.

  11. GNUNotUnix says:

    I need to find a therapist, and I trust SSC readers the most. I’ve had a therapist once, but she it was only for a few months, and I found her by simply showing up looking for help at my university’s mental health center. I’m not a student any more so that’s not an option. That therapist was useful to me but not for very long. I happened to lose confidence in her because of some of her beliefs about my early childhood etc… I would like to find a therapist I can trust and respect, and if Scott was doing therapy near me, I’d sign up in a heartbeat.

    Does anyone here have any advice or stories about how to find a therapist? I realize it’s going to be a process of trying several therapists until I find one I like, but is there some way to gain an edge in this process? Any help would be appreciated!

    • Machina ex Deus says:

      I had a good therapist, followed by a ten-year break for college, first job, etc. When I realized I should go see a therapist again, I called my old therapist for recommendations or referrals. He didn’t know anyone in my current location, but he knew someone who did. That’s how I got my second good therapist.

      Scott has a post from three years ago titled Getting a Therapist, which has links to pages he thought gave useful advice, as well as a handful of specifically-recommended-by-readers therapists.

      Finding a good prescribing psychiatrist… I’ve had much worse luck there. I now settle for someone who’s at least efficient about getting through the med-check conversation. (Scott: is there anything in psychiatry more aggravatingly worthless than a med-check visit?)

  12. random832 says:

    I can’t find it now, but there was either an essay or a comment either here or on some similar site like less wrong that I remember reading, with an extended analogy involving something two competing ice cream stands both in the middle of a beach, even though each would make more profit if they located themselves at either end. Does this ring a bell for anyone else?

  13. CatCube says:

    Just got the e-mail from NCEES! Passed the PE exam. I am so glad I don’t have to face another 3 months of studying and an 8-hour exam.

    • keranih says:

      *high five!*

      (Do you have to take refresher or CE courses from time to time?)

      • CatCube says:

        30 Professional Development Hours (PDHs) are required every two years prior to renewing registration. I don’t know that I’ll have a whole lot of trouble with that in my current job, as training classes aren’t uncommon. I’m doing an 80-hour course on bridge inspection at the end of June, for example.

  14. John Schilling says:

    OK, logging in to the site. I accept the requirement as reasonable and necessary. The no-bots captcha, ditto. A no-bots captcha based on a simple math problem presented in unobscured plain text, seems weak but maybe that’s all we really need. Fine. But the request I just got was, literally:

    “Prove you’re human: 7 + 7 = ___”

    Prove I’m human? With computers now besting humans at chess, go, and even Jeopardy, I’m supposed to prove my humanity with that last bastion of meaty supremacy, arithmetic?

    • random832 says:

      You’re proving your ability to read the form, not to do the math. The form will be changed if sufficiently many spambot programs are taught how to recognize that form.

      Sites that make for juicier targets have more sophisticated spambot detection mechanisms, to avoid the scenario of a human checking the form daily to see how it has changed. AIUI google’s “recaptcha” includes secret sauce that measures input timings in addition to your actual responses to the image recognition prompts.

    • Rob K says:

      “Prove you’re human. In 1954, Willie Mays, in an emphatic stroke of Byzantine whimsy, made his over-the-shoulder catch off of Vic Wertz. What was it not unlike?”

      • Deiseach says:

        “Prove you’re human. In 1954, Willie Mays, in an emphatic stroke of Byzantine whimsy, made his over-the-shoulder catch off of Vic Wertz. What was it not unlike?”

        This? 🙂

        It isn’t so quiet now boy.” Ring’s riposte to Limerick’s Donal Broderick after scoring three goals in four minutes to win the 1956 Munster final. Broderick had held Ring scoreless for more than fifty minutes and had commented that he was having a quiet day.

        • Rob K says:

          I’m sorry, the correct answer was “it was not unlike watching Atlantis rise again from the sea, the bones of her kings new covered with flesh.”

  15. The original Mr. X says:

    I have a question about the Agricultural and Industrial Revolutions. One of the factors commonly given for the Industrial Revolution was the surplus population created due to 18th-century agricultural advances, which freed up a large workforce for the big new factories being built. Generally, however, population surpluses seem to have the opposite effect: a large supply of workers meant lower wages, decreasing the cost of hiring people and disincentivising technological advancement. So why wasn’t this the case in the Industrial Revolution? Or if the role of increasing population in the Industrial Revolution over-hyped?

    • Anonymous says:

      If you ask Clark, it was because upper class ihatei became mainstream, replacing the former, less wellbred inhabitants of the lower strata.

    • herbert herberson says:

      decreasing the cost of hiring people and disincentivising technological advancement

      What makes you say these are linked? If I’m managing an artisanal industry, my main incentive is to get the most highly skilled workers. But if there’s a huge pool of cheap unskilled labor available, I now have an alternative option of investing in more technologically advanced capital which does not require those artisanal skills.

      • The original Mr. X says:

        I say these are linked because this is the argument I’ve most often seen given for why Ming China or ancient Rome didn’t industrialise, or conversely why Europe saw more innovation and technological advancement after the Black Death than before it.

    • Salem says:

      You’re modelling it wrong.

      Yes, a one-off population increase with no other change is going to lower wages and reduce returns to capital investment. But that’s not what happened.

      What actually happened is an increase in productivity (i.e. the Agricultural Revolution). This:
      a) raised wages.
      b) raised the price of land.
      c) As a result of a), the population increased.

      So the population is larger, output is higher, and wages are higher (but sure, not as high as they would have been had the population not increased). This leads to three things:

      1) Because output is higher, existing markets are larger, so deeper capital investments become attractive (maybe your canal isn’t profitable at 20 barges per day but is profitable at 40). As Adam Smith said, the division of labour (and hence capital investment) is limited by the size of the market.
      2) Because the population is larger and wages are higher, wider capital investments become attractive (maybe your linen factory isn’t profitable if only 10 people are buying shirts a day, but is profitable at 20).
      3) Because wages are higher, substituting capital for labour becomes more attractive.

      This caused huge amounts of capital investments to be made (and note that (1) and (2) further increase wages, although (3) lowers them). Hence the industrial revolution.

      Note this has nothing to do with “surplus population.” That sort of crude Marxian/Malthusian thinking doesn’t explain anything.

    • Deiseach says:

      Generally, however, population surpluses seem to have the opposite effect: a large supply of workers meant lower wages, decreasing the cost of hiring people and disincentivising technological advancement.

      It’s not a case of “all the workers used to be farm labourers and now they are factory workers”, it’s “about the same amount of farm labourers are still there, but now the excess population can be absorbed by new jobs in the towns”. Without the new factory jobs, the excess workers would have little choice but to stay on the land; now that there were new jobs opening up, they had an outlet.

      Also, farm work is very seasonal. Wages increased for periods like harvest time and dropped down again in quiet periods such as winter, besides that some farmers would have paid their workers ‘in kind’ (providing food and beer etc.) and so money wages would have been smaller. Industrial wages were constant.

      Thirdly, during the period of the late 18th/early 19th century, you had the Revolutionary War in America and then the Napoleonic Wars. This drove up farm wages as there was increased demand for farm goods (an army marches on its stomach, after all) and this in turn meant more money for wages, as well as men going into the army and helping absorb some of that surplus population. So industrial employers were competing with farm owners for workers and had to pay a reasonably attractive wage. (Contrariwise, after the wars, farm labourer wages fell again).

      And early industries were labour intensive, even if they were seen as taking jobs away from men. Women and children made up a substantial proportion of the workforce in textile mills (indeed, at the height of the mills up to the 1920s, women were the majority of the workforce; in Northern Ireland men worked in the shipyards and women worked in the linen mills).

    • yodelyak says:

      This is from an email I shared w/ a friend back in 2012. It seems to relate to industrial revolution in England, and why it happened there, but seemingly similar circumstances elsewhere didn’t also cause industrial revolutions:

      “I’d recommend The Great Divergence, by Kenneth Pomeranz…

      The Great Divergence makes two claims, the first of which I found fairly novel, the second less so. The first is that up to as late as 1750, the leading economy in the East was on just about every available metric the more sophisticated, compared to the most sophisticated economy in the West (whether Britain taken singly or a broader chunk of Europe taken as a whole). Areas of China were integrated across larger populations, benefited from greater specialization, and overall produced a larger per-capita output than any of the economies of England or anywhere in the West.

      TGD’s second claim is that what stood primarily in the way of most economies–China’s, and those in Europe too, was the problem that technological improvements were labor-saving rather than land-saving, so that economies hit solid ceilings in terms of what could be produced given a limited land holding–and that population density, although it allowed for a greater specialization, hit firm limits because of this. As TGD has it, what created the boom for England, and Europe more broadly, was the sudden erasure of this land limitation, resulting from the colonies, the slave trade, and of course, the advent of fossil fuels. First the American colonies, even after the revolution, provided lumber, sugar, and cotton to match industrial capacity even as capacity grew ten-fold, and then as the civil war shut down some (but not all) American markets, India and the Indies provided the cotton, as well as another massive increase in lumber and so on.

      TGD makes no attempt to examine any personalities involved. There is no talk of Rockefellers, Nobels, or Rothschilds or etc. I think I prefer no treatment to poor treatment, and I see poor treatment so often that I didn’t really mind the omission. The book is nonetheless too long, because sentence after sentence presents price, population, or other comparisons between the West and the East at different times. The book would really be improved by the inclusion of a couple more good tables, and the removal of the related excess verbiage.”

      • cassander says:

        >First the American colonies, even after the revolution, provided lumber, sugar, and cotton to match industrial capacity even as capacity grew ten-fold, and then as the civil war shut down some (but not all) American markets, India and the Indies provided the cotton, as well as another massive increase in lumber and so on.

        The trouble with this theory is that lumber, sugar, and cotton were all luxury goods. The lack of them was not a hard limit on any pre-modern economy, the lack of food was, and the colonies did not feed the home countries. in the 18th century. If anything, it was the reverse.

        Now, you could argue that colonial land being used for luxury products meant that home land could turned towards food production, but the opposite of that was actually happening. Moreover, any theory of industrial revolution predicated on the assumption that colonialism was a cause not a consequence should predict that spain, not the UK, should have been the birthplace of the industrial revolution.

        • The original Mr. X says:

          Moreover, any theory of industrial revolution predicated on the assumption that colonialism was a cause not a consequence should predict that spain, not the UK, should have been the birthplace of the industrial revolution.

          Only if it predicts that colonialism was a sufficient cause. If colonialism was necessary but not sufficient for industrialisation, there’s nothing surprising about the fact that some colonial countries industrialised and others didn’t.

  16. cassander says:

    So necromancers are interesting. Lots of interesting story angles on them, but there seems to be a dearth of fantasy out there that uses them as anything other than generic villains. Not a fan of urban fantasy, and anything young adult is almost certainly out, but but almost anything else goes, though my preferences run to the darker side of things. The Prince/King/Emperor of Thorns, for example, I I absolutely loved. Ditto the black company, which while not exactly necromancers, does have a fun take on wizards of all sorts. Old Kingdom trilogy was also interesting.

    Does anyone have any recommendations?

    • engleberg says:

      I liked the good guy elf necromancers in Eberron, but it’s an rpg not a novel. I loved Saberhagen’s Vlad the Impaler good guy. The Vampire Tapes set a standard no vampire novel before or since has even approached. The rest of the series was good too.

    • Trofim_Lysenko says:

      Max Gladstone’s Craft Sequence, and especially the first book in the series, Three Parts Dead. Decades after a war between the Craftsmen and Craftswomen of the world and the Gods and their priesthoods, souls and fractions thereof are the fundamental unit of the economy, which means that there is little fundamental difference between a necromancer, a lawyer, and an investment banker/financial analyst.

      In the first book, one of the last independent Old Gods of Fire has suddenly and catastrophically died, and a senior Necromantic partner from an established Craft Firm along with her novice assistant are called in by the Church in order to handle the resurrection/restructuring.

      Note: publication order doesn’t match chronological order, but most of the stories are geographically separate and set years or decades apart, making them relatively stand-alone, so you can read them in either order. My only advice would be to read First Last Snow before Two Serpents Rise (books 1 and 2 chronologically, books 4 and 2 in publication order. You may’ve noticed the naming gimmick allows you to order the books chronologically: First Last Snow, Two Serpents Rise, Three Parts Dead, Four Roads Cross, Full Fathom Five.

    • John Schilling says:

      For a very dark take on necromancy, David Brin’s “Thor Meets Captain America“, later adapted in graphic-novel form as The Life Eaters. Set in an alternate World War II, so may not meet your “no urban fantasy” requirement, but very good. No prize for guessing who has necromancers on staff in an alternate WWII, or how they get the raw material.

    • Skivverus says:

      Well, “Chronicles of the Necromancer” probably fits the bill; relatively generic (though enjoyable) fantasy other than the abilities of the titular character, though.

    • AeXeaz says:

      Brian McNaughton’s The Throne of Bones is a collection of dark, cynical, funny stories about necromancers and ghouls.

  17. RhetoricalViking says:

    I’m writing a blog post and would like to link to the Consequentialism FAQ. Is there anywhere that’s still posted? I was a lapsed consequentalist until I read that, and would love to see that up again.

    • Evan Þ says:

      The Internet Archive still has it.

      (Unfortunately, the actual raikoth dot net seems to have been taken over by malware, which is really unfortunate for people following old inbound links.)

      • Anon. says:

        That piece seems absurdly weak to me. Everything rests on “moral intuitions”, which any self-respecting rationalist should just dismiss out of hand. Not only is there no evidence that our intuitions are reliable guides to the truth, there’s plenty of evidence that they are deliberately misleading: all those behavioral biases, heuristics, and nonveridical perceptions evolved for a reason.

        You wouldn’t let your folkbiological intuitions override scientific results in biology, so why would you give a flying fuck about your moral intuitions, let alone use them as the foundation for your entire system?

        • Dabbler says:

          Although I don’t agree with the piece, I don’t think moral intuitions should be dismissed so easily.

          Question- What rational justification is there for being moral? Whatever justification you give it? All sorts of philosophical attempts have been made.

          If it is a question of pragmatism in any way, then there is no reason to obey the moral rules when they aren’t in your self interest. So don’t appeal to that.

          The only possible solution is that for some reason you want to be moral, even though you can’t rationally justify it. Which means either arbitrary choices or moral intuitions.

          • Anon. says:

            I think “I want to be moral” is a fairly strange terminal value. Not obvious what advantage it confers. Therefore, the genealogical approach is useful here. Instead of accepting it, first ask: “why do I have this value? What is its source, how has this meme spread? What purpose does it serve?”

          • Dabbler says:

            You are making several fallacies here.

            First, I can know perfectly well that something would be evolutionary advantageous and NOT want to do it. Evolutionary purpose is not necessarily our purpose. See http://lesswrong.com/lw/l1/evolutionary_psychology/ for a good and brief explanation.

            Second, that still doesn’t help you define what “moral” is. That can only be a philosophical question.

          • Anon. says:

            First, I can know perfectly well that something would be evolutionary advantageous and NOT want to do it.

            I think this is a point for my side.

            “Why do I have this value?”
            “Because it was evolutionarily advantageous.”

            But as you say this is not sufficient, so we can drop it as a terminal value.

          • Dabbler says:

            Isn’t the entire point implicit in your argument to try and view morality from an evolutionary perspective? And, like all those who view morality from an evolutionary perspective, to solve moral debates by appeals to evolution?

            Maybe what we’re having is a terminology problem.

            “Moral value”, as I use it, is a value, terminal or not, that is part of a broader code of conduct, and a value which is based on something other than self-interest.

            I think you are using “terminal value” to mean something ultimately valued for its own sake.

            If a person should have a moral value (and I don’t think there is a rational answer to those who do not want to have them), how are they going to decide what moral values they want? By, assuming I understand your terms right, showing how they ultimately trace back to terminal values. There is no other way.

            The better uses of “moral intuitions” are effectively saying that people do value certain things, and then trying to take these things to their logical conclusions. Not all uses are like this, but some are. So I don’t like throwing out the concept entirely.

          • Anon. says:

            Isn’t the entire point implicit in your argument to try and view morality from an evolutionary perspective?

            Yes.

            And, like all those who view morality from an evolutionary perspective, to solve moral debates by appeals to evolution?

            Well, in a way. My argument is not “we evolved to do X, therefore we ought to do X”. It is that once you understand the source of your desires around X, we can solve the question of “for some reason you want to be moral”. The answer is not using your intuitions to find the best way of being moral. It’s not wanting to be moral. The reason is that, as you said, “evolved purpose is not necessarily our purpose”. Just because you evolved to want to be moral doesn’t mean you should.

          • Dabbler says:

            What can “should” possibly mean, except in the context of a moral code?

        • Paul Zrimsek says:

          Which scientific results in ethics are the intuitionists letting their intuitions override?

          • Anon. says:

            If they are realists (“morality cannot just be some ghostly law existing solely in the metaphysical realm”), they ignore that there is an absence of evidence for realism (and Absence of Evidence Is Evidence of Absence). I don’t see how you could possibly square the Consequentialism FAQ with correspondence theory of truth, which I believe Scott subscribes to.

            The intuitionist might now retreat to “we don’t need external evidence, our intuitions of right and wrong are the evidence”. But this is wrong: 1) mutually exclusive intuitions between different people, 2) the criticism of intuitions as reliable guides to the truth, above.

            It also ignores the much more parsimonious explanation of morality as an evolved mechanism.

          • Paul Zrimsek says:

            In which case you might want to go back and try with a different analogy– unless you believe that biology is a pseudo-study which people have evolved to believe in anyway.

          • Anon. says:

            I don’t follow.

            Folkbiology is the pseudo-study that people have evolved to believe in. Actual biology supplants it and contradicts it.

          • Paul Zrimsek says:

            See, that’s just where you lose me: scientific biology and folk biology differ only in their methods, not in their subject matter. Scientific biology, like folk biology, is the study of living things– not the study of how people come to have the delusion that there are living things. I can’t tell whether your target here is just intuitionism, or realism tout court; only that whatever position you hold, you’re very, very confident in it.

        • rlms says:

          The difference is that folkbiological intuitions are descriptive, moral intuitions are normative. How do you propose people distinguish between moral systems other than by how well they match intuitions? Nihilism and theism are possible options, but other than those I can’t think of anything.

        • Philosophisticat says:

          Speaking as a philosopher of ethics, there is a forceful argument along the lines you mention for distrusting moral intuitions (it goes by the name of the evolutionary debunking argument and is is a major and controversial topic in contemporary metaethics – for a primer and some references, see https://plato.stanford.edu/entries/morality-biology/#EvoBioDebMor), but you are coming across as massively overconfident to the point of arrogance. It’s a bad look. Be like Socrates.

  18. Dabbler says:

    On another topic- regarding Pope Francis and what he considers to be true, do people consider him to be intellectually consistent with himself? I would have said obviously not given his stated beliefs (including the ones that are not formal doctrine!) and how they fit together, especially with the Catholic doctrine he claims to still uphold. But I am curious to see if others think otherwise.

    • Conrad Honcho says:

      Aside from amoris laetitia, what “stated beliefs” of Francis contradict Catholic doctrine? Specific examples, please.

      ETA: I think what you’ll find is that the things you think Francis said that were contrary to Catholic doctrine were either 1) misreported or 2) you and reporters misunderstood Catholic doctrine.

      The most famous example of this is “who am I to judge?” referring to homosexuals who come to confession. The media reported this as “gay is okay! Gay marriage and all kinds of non-procreative sexual acts now Catholic-approved!” Because that’s what they want to hear. And they think Catholic doctrine up until this point was “God hates f_gs.”

      None of this is true. Defining someone by their sexuality and putting them in a neat little box is a modernist idea. In the Catholic worldview, people are complex, and sexuality is complex. A person is not “a homosexual.” A person is a person, with a name, and a soul, lots of different desires. Some people experience same-sex attraction. This is not a sin, and being a person with same-sex attraction (or opposite sex attraction, or desire to masturbate, or any of those things) is not a sin. There’s no sin you commit just by existing. Now disordered sexual acts, like masturbating, sex outside of marriage, gay sex, those are sins. Being gay isn’t a sin. Having gay sex is a sin.

      So when asked how Francis would deal with a gay person coming to confession, “who am I to judge” refers to the same-sex attraction, not the same-sex actions. This is in keeping with the Catholic doctrine of treating people with same sex desires (and all people, really) with compassion and not marginalizing them. Your Kink Is Not My Kink But You Still Shouldn’t Act On Your Kink.

      So the whole “Francis says weird stuff that’s totally overturning Catholic doctrine!” narrative is really created by the media not understanding Catholic doctrine to begin with, and then hearing whatever they want to hear in anything Francis says, and then misreporting both.

      The reason Catholics are at times annoyed with Francis is because:

      1) he could perhaps be more careful with the slimy media to avoid these sorts of annoyances.

      2) he’s sticking his nose into world politics, informed primarily by his South American quasi-socialist political beliefs, none of which have anything to do with Christ’s Church.

      3) if you’re interested in internal Church politics, he’s favoring cardinals and bishops who are down with item #2 up there.

      But little of this has anything to do with the type of “I’m an atheist but man Pope Francis is really great!” comments you find all over reddit.

      • Eric Rall says:

        My understanding is that a lot of the perceived disconnect is a consequence of people with a Protestant cultural background trying to understand Catholic doctrine without understanding the differences in Protestant vs Catholic conceptions of sin.

        To oversimplify, Protestants tend to think of sinning as being like committing a crime: there are clear rules that a good person should be able to avoid breaking, and breaking the rules is a really big deal; but Catholics think of sin more in terms of falling short of an ideal of perfect thought and behavior. To Catholics, sin is a pervasive feature of the human condition, and the important thing is to recognize your failings and continuously work towards improving yourself.

        From a Protestant perspective, the panoply of things Catholics consider sinful seem like overbearing strictures, and the readiness of forgiveness for sin (which can be uncharitably oversimplified as “you can do anything you want as long as you feel guilty about it afterwards”) looks like hypocrisy.

        Source: I was raised Episcopalian (technically Protestant, but has a number of “Catholicism Light” features), my father was raised Catholic, and I’ve taken an interest in theology and comparative religion as an offshoot of my interest in history.

        • Conrad Honcho says:

          I agree with that. A similar distinction is in the nature of the Bible. In the Protestant worldview, the rules are contained in the Bible, so they read the Bible. In the Catholic worldview, the Catholic philosophy is central, and the Bible was compiled by the early church fathers as a liturgical tool to teach that philosophy. Catholics don’t read the Bible because Catholics “wrote” the Bible.

          It’s the difference between understanding physics and reading a physics book. The rules came from physics and are described as best physicists can in the physics book. But the book is not physics, cannot ever contain all of physics, and physics is still being worked out by physicists who generally agree they will never understand the totality of physics.

          Source: Raised Catholic, became atheist because I thought I was so smart, got really mad at the world and started writing a manifesto about how to fix it and then realized I was basically rewriting the gospels and oooohhhhhhhhh I get it now and went back to being Catholic.

          • Dabbler says:

            Thank you for answering. Although as somebody raised Catholic- I have never heard such things outside the Internet. There are clearly conservative Catholics who are far more rules-oriented than those you describe.

          • The original Mr. X says:

            as somebody raised Catholic

            Unfortunately, given the frankly shite state of catechesis in most Catholic parishes nowadays, that doesn’t mean much.

          • Dabbler says:

            True enough. But this also doesn’t explain, for instance, why the dubia happened at all if what Francis is saying is how Catholics have thought all along.

          • Conrad Honcho says:

            @Dabbler

            Yes, that’s why I started with “aside from amoris laetitia.”

            In my CCD classes growing up, yeah we didn’t get a lot of theology. It was much more rules based. I think part of coming into ones own as a Catholic believer as an adult is failing to understand the purpose of the rules, dismissing them as arbitrary, and then gaining life experience to understand how the rules were derived in the first place. Then it all kind of falls into place and “clicks.”

            For similar but non-Catholic explorations into this, you might consider reading C.S. Lewis’ Mere Christianity, or watching Jordan Peterson’s Maps of Meaning lecture videos, or the new thing he’s doing focused solely on the Bible. I know it’s shitty to say “here, watch 25 hours of videos,” but the evolution of theology is complicated. At least they’re entertaining.

        • The original Mr. X says:

          To oversimplify, Protestants tend to think of sinning as being like committing a crime: there are clear rules that a good person should be able to avoid breaking, and breaking the rules is a really big deal;

          I don’t know about that; Luther and Calvin were pretty big on the idea of total depravity, and I’ve known a few Evangelical Protestants who’d say things like “Human nature is inherently sinful”.

  19. Dabbler says:

    Requesting advice. People around me seem to be viewing me as a manchild, incapable of full adulthood. I want to try and persuade them that I should be considered fully adult. I’m 25 and trying to get it right, but it’s very hard to even see why people refuse.

    EDIT:
    I am autistic (originally diagnosed with Aspergers Syndrome). I have gone No Contact with my parents because they told me that they would never consider me an adult no matter what I did, and were both putting severe pressure on me to give up on everything I tried to do to become a full adult and refusing to give me any support whatsoever in fixing myself. The people I am trying to deal with are mostly friends of my parents, because they are a lot of my social group including my boss.

    I try to ask people why they aren’t treating me as a full adult, and what I am doing wrong. Most people give obfuscating answers, while one family friend (Dave Keyoe) outright stated that I should give up on trying to be a full adult, and redefine “adult” based on my disability. My grandfather claimed that as an autistic I could never date or drive.

    That being said, I am hardly without fault in all this.

    Things I can think of that I did wrong: (Possible)
    -I mis-budgeted, and ended up about $3000 dollars in debt on driving lessons. My parents had refused to transfer my rent payments to myself despite repeated arguments for two months (about $2000 of it), the other one was a student loan to make a rent payment once they finally did.
    But everyone keeps insisting this is normal,
    and puts pressure on me to spend more.

    -Out of excessive stress, I offered to restore contact with my parents IF they were to help me get a driving license (big mistake!). They kept not being available for the date my test was on, and those who knew kept pressuring me to take them rather than the proper date.

    But family friends who I know for a fact do not know this do the exact same things.

    -I sent terms of reconciliation to an intermediary (C.O.B), basically demanding my parents accept responsibility and that I disown them from a parental role that they had abused, asking as a courtesy for her thoughts before I sent them. When she was obstructionist, I requested (since she had agreed to act as intermediary) that she send them. When she refused, I told her that if I didn’t have an intermediary I wouldn’t bother trying to negotiate. She insulted me, I told her I was going minimal contact with her, she cut off contact with me.
    As above.

    -I tend to hit myself in frustration, if rarely around others.
    But only one family friend knows I still do this.
    They claim it has nothing to do with adulthood.

    Apart from that, I have nothing.

    ————-
    People keep putting pressure on me to accept free help, such as loans and free driving lessons. I can’t take it for several reasons:

    1- In an environment where people are either implying or outright stating they see me as a manchild because I’m autistic, I don’t want to show any sign of weakness. (I screwed this up, but I figure any more signs would make it worse)
    2- I am trying to be financially responsible. This is impossible if I am living off loans and gifts, as people are constantly pressuring me to.

    ——————

    The question is: What do I do now? I understand about the driver’s license but I don’t understand why people outright put pressure on me not to be fiscally responsible!

    • yodelyak says:

      Hi Dabbler,

      First, I’m always sad when I think about people who are estranged from their parents or other family for any reason. Having to go ‘no contact’ with your parents sounds like a serious loss.

      I expect you’ll get much good advice from people with more direct experience with autism; I have basically none. Still, you might find my perspective helpful, because it doesn’t seem to be what you expect at all. The short version is, have you looked into whether your physical appearance or body language or etc. might contain cues that you are not “with it” in terms of having a mind that is firmly in control of how you present your body? (Appearance here includes how you dress and whether your hair looks good, but also and more importantly, whether you stutter or have other verbal tics, or have a really unusual posture, or otherwise have anything about your person that seems out of control in a way that most people put a large amount of effort–at least until it becomes habitual–into keeping under control as a matter of keeping up appearances.) Something a family member of mine did well into her late twenties was end all of her sentences with an upward lilt, so she always sounded like she was asking questions or seeking affirmation.

      I said I have no experience with autism. That’s no exaggeration; I have no experience with autism outside some general cultural literacy, e.g. having read “The Curious Case of the Dog in the Night-Time,” and knowing some people who have direct experience with autism and who have talked about it, such as a family member who works with groups of children that sometimes include an individual or two with autism. What I do have is a lot of experience in field politics and face-to-face persuasion; my suggestion would be just as applicable if you hadn’t identified yourself as an individual with autism, but only as a 25-year-old with challenges getting other adults to treat you like an adult.

      One specific experience I had that seems relevant is that I used to hire people and train them to work as face-to-face fundraisers or political persuasion canvassers. In that line of work I developed very good intuitions–I can make successful predictions–about which of the folks I was hiring would be highly successful, and which less so, just off a 1- or 2-minute interaction with a person, and often my predictions turned on pretty superficial questions of a person’s control over their appearance and body language.

      For one really simple example of an intuition I developed, I noticed early on that people whose bodies are visibly females, when going door-to-door to talk to voters, find 5 to 10% more people at home than men do. Because I am male, and the effect extends to me, and because I had enough data to observe a remarkable consistency in terms of how women have a higher number of people coming to the door than men.

      That intuition about gender was kind of exceptional, in that it turned on something relatively basic and immutable. Many of my intuitions had more to do with “this person seems not to have successfully seized control of basic parts of how they present themselves, so they will at a minimum need more training than others before they can deliver a precision message on behalf of this campaign.” Some of my starting intuitions proved really ill-founded (being dressed formally doesn’t help canvassers get more or better conversations, possibly because they look like religious evangelists, and then when they do get conversations are uncomfortable and stiff in their bearing) but other intuitions were borne out (it got so I could recognize in interviews the kind of interpersonal style that would bring back a vastly disproportionate number of “undecideds” because people are more likely to say ‘undecided’ if the person asking them does so in a half-hearted, sorry-to-bother-you-miss kind of way.) Anyway, now that I’ve spend too long writing about how much this stuff really does make a difference, what I really want to impress you with is that most of the things that distinguish barely acceptable door-to-door “get out the vote” canvassers from the best fundraising canvassers in the country are totally trainable: eye contact, confidence, body positioning that is relaxed and nonthreatening, speaking loud enough that you are heard, but not loud enough to be shouting, and varying your tone and volume to create cadence and reflect your feelings. Something you might try looking for is a toast masters or an optimists club or somewhere you can go to interact with strangers who make a point of trying to out-adult each other in terms of their ability to stand up and present their view in a way that manifests their smarts where people can see them. (If you are trying to reason with people verbally about the extent to which they should treat you like an adult, rather than simply *showing* them with your body language, that’s already quite outside normal adult behavior.) This is an area where I think just about anyone can improve, so it’s a pretty good investment for anyone to take another look at what they can do to boost their interpersonal effectiveness. It is probably worth also saying that while almost everyone can improve, and even level up once or twice, almost everybody can also expect to always have to deal with people who are a level or two or three higher than them, so it’s a good idea not to make too big a habit of comparing yourself to others for purposes of self-worth. Everyone has different talents and etc.

      Sorry this comment is disjointed. I’m adhd myself, and today is one of the worst days I’ve had in a while. Hope this was helpful.

      • Dabbler says:

        Hi yodelyak,

        Thank you for helping.

        Your reasoning seems to make a lot of sense, but I’m mystified as to what body language mistakes I could be making. Nobody has mentioned anything in particular that is wrong, I have dressed reasonably as far as I know, and I have undergone plenty of social skills training. I would consider washing my clothes more often, but I’m strapped for cash as it is. And for a little while at least one my relatives actually treated me with respect.

        Maybe you could give me some good general guidelines? How to dress, how to act, what to say etc? What sort of general attitude would make people think that I am an independent adult and that intervention in my life is not necessary?

        Would you have any idea why any of this would lead people to actually put pressure on me not to be fiscally responsible? I don’t see how this links into any of the above.

        Ed

        • Eric Rall says:

          Yodelyak mentioned Toastmasters in passing, and I’d like to call them out in more detail as a tool for improving body language and speech mannerisms. They’re a network of clubs of people at various levels of experience who are trying to improve their public speaking and related skills, with an organization behind them that provides a good framework of processes and course materials. The general format is for club members to take turns making a variety of speeches (a mix of prepared speeches and impromtu statements) to the rest of the club, and for the rest of the club to give you constructive feedback for what you did well and what you can work on for next time.

          If you’re doing something “wrong” in terms of how you present yourself, a forum like that can be a very good place to get feedback and suggestions. They can do a lot more for you than we can, since we can’t see your body language from here.

          • Dabbler says:

            I’ll look up ToastMasters. Although I did have a social skills instructor before and it didn’t help much. I was less sociable with others than with her admittedly, but she never treated me as an adult (even if she did apologise for the egregious offence that springs to mind six months later).

        • CatCube says:

          Part of nobody mentioning anything “wrong” in what you might be doing is that giving somebody criticism of their body language or mannerisms is often considered extremely rude. Also, many people do not take it well when they’re criticized, considering it a personal attack (and they’re sometimes right). These factors mean that most people find it very, very, awkward to have this conversation, and generally avoid it.

          Of course, this results in everybody talking to each other about how somebody has terrible mannerisms, or smells awful, but nobody telling the person under discussion.

          Except for people who don’t mind being assholes or bullies, which can result in the subject discounting the criticism and digging in. (This isn’t to say that just because somebody is saying it means it’s true–they could be lying to cut somebody down; but they *might* be saying something to someone’s face that everybody else is saying behind their back.)

          In the adult world, though, even bullies will rarely target even truthful poor mannerisms and the like; that’s really a high-school thing, and everybody else will see it that way. Unfortunately, what often happens is that the person with the poor mannerisms will be quietly shuffled off into the corner, then let go at the earliest available opportunity.

        • yodelyak says:

          Hi Ed,

          It is really, really hard to give *specific* advice on this to someone I haven’t met. And the first piece of advice I usually give anyone who asks for advice that seems like it might be of high importance, even someone I know, is that you should treat advice as information, not a recipe and definitely not as instructions. I should have started with that from the get-go. You know what I said, given what you said, so you have more information than you did before… but it may be low-quality or even negative quality information, (garbage in, garbage out, for one) and unless I were your parents or other life-long acquaintance, I am highly unlikely to know as much about your situation as you do. What happens after you listen to what I say is your responsibility, not mine, and that’s just as true whether you do exactly what I say, or exactly the opposite. That’s the breaks, but that’s the world.

          So, three thoughts.

          I. I agree the connection with fiscal responsibility doesn’t directly connect with my experience–it’s oddly specific, if you’re just coming across as juvenile or strange–and I may be completely off-base with suggesting that your situation may have to do with something other than your relatives perceiving actual fiscal irresponsibility on your part. The obvious piece of advice to someone who is frequently making pleas before judges is not “improve your self-presentation when you are in court” but “stop breaking the law”; I have taken your word on your description of your financial management, but really it’s perfectly reasonable for me to side with your relatives instead of you, simply because you report that this is a running theme, so the weight of the evidence might seem to be with them.

          In your mid-twenties, it seems pretty unusual to be getting multiple adults outside your parents stressing fiscal responsibility; my experience was that everyone I knew, of any age, was so busy with their own problems that they didn’t have time to instruct me on mine. One thing you can sometimes infer when you get a lot of unsolicited advice is that you have a lot of insecure people in your life who are giving advice because it gives them the feeling of being in the know. That’s certainly a feeling I’m experiencing as I write this comment, and it’s generally a pleasant feeling. They also may perceive the financial risks you take to have some nonzero chance of being risks with their money, so it isn’t really that odd they might be pretty likely to want to see you living extremely conservatively. They are exposed to the downside, but not the upside. If that’s right, one thing you could try is demonstrating enough adulthood for your relatives to perceive themselves as exposed to upsides, not just downsides, of your financial risk-taking. So, if you experience financial success, maybe try making an appropriate present (a nice game for their child, perhaps, or having people over to your residence and cooking a nicer meal for them)? I don’t know what your resource level is, so you can imagine how impossible it is for me to give good advice here.

          II. Going back to the subject of your personal presentation, I don’t know if you come across like a nerdy Bill Gates or like Andre the Giant’s character in The Princess Bride or like Peter Pan or like Stephen Hawking as a young man or like Stephen Hawking as an old man or or or or… it’s just really not possible for me to help much there.

          Instead, I think it makes sense for you to focus on 1. getting *honest* in-person feedback from people who are forming a fresh or even a first impression, and 2. on getting *positive* feedback from people who can help you keep your energy and spirits up as you identify things about yourself you want to work on. Depending on your risk profile, you might want more of #1 (which will push you to try many self-improvements) or of #2 (which will help you avoid forming a lastingly negative self-image or end up worse than you started because you are getting discouraged or despairing or just plain weird, all of which can sometimes happen to people who spend too much time policing how they appear and striving to be liked rather than just kickin’ it with people who already like them.)

          III. I don’t want to over-emphasize this, but you might also think about the idea of a self-fulfilling prophecy, and whether you might be the source of your own problem. Something I ran into in law school were some people whose own low expectations for social success became self-fulfilling prophecies when they defensively vocalized them. The worst example of this was a gal who I saw introduce herself, as a matter of first impression, to the 30 other law students who would be in all her 1st-year classes as “Someone with no friends, someone who always gets bullied and I hope none of you are going to bully me.” That of course meant people who want to be friends with the people who are cool knew she wasn’t their top choice for friend material… which since almost no one is entirely immune to wanting to hang out with the cool crowd, was a big handicap right out the gate. Plus, some people (especially some men in law school) know they are sometimes viewed as too aggressive/competitive, and would avoid her rather than potentially get her complaining about them. In my case, I’m a highly conscientious person, and am very a.d.d. sometimes, and can come across extremely intense in person, and have a high tendency to generate false impressions that I’m flirting… and I too didn’t want to hang out with her because I was reluctant to hang out with this girl who was obviously feeling vulnerable and might come on pretty strong if I was too nice, which I almost certainly would be. Plus, law students are smart enough that we all know this kind of self-fulfilling prophetic behavior is really damaging to one’s social status, at least for the reason that everyone wants to hang with the cool kids, and she said it out loud in front of all of us, so it was true common knowledge that she was the sort of person who’d handicap herself in this way, for no reason, as a kind of immediate own-goal. That in turn meant that even if all thirty students later interacted with her and realized we liked her we might each still think we were the only people who did. Luckily just about every group of people has a few people whose social strategy is to be the person who likes everyone, openly, so this kind of thing may tend to get worn down over time, somewhat, but it’s still not great, especially among law students who can be… insecure and Machiavellian.

          If there are dynamics like this working against you, where your relatives are inferring you are not financially responsible because you are defensive about being financially responsible, it might be a good idea to look for a way to make a clean transition which your relatives will all infer all of them can see happen. Something like switching to a new geography may be overkill and then some, but something like privately telling one generally complimentary family member about how nice it was to get a raise at work, and how much you are saving, probably won’t get the job done. Something like hosting a tasteful dinner to celebrate a financial milestone, and making sure even the relatives who aren’t there know that lots of relatives were there and talking about it… that might make the most sense.

          • Dabbler says:

            I:
            Everyone around me seems to treat being fiscally dependent at my age as normal. They have outright said it is normal. That’s what so confusing!

            II:
            Thank you. I think I know some people I can ask. I’ll try that.

            III:
            I tried taking people to dinner. The event worked well in the short term with my sister and her husband, but things quickly went downhill after that.

            It’s probably too late now. Everyone knows I’m a few thousand dollars in debt to my parents. Most people know that I was defrauded to the tune of $3000+ by a bad driving instructor.

            You might very well be right. Though it doesn’t explain why repeated demands to be taught what I was doing wrong so I could fix it failed.

            But I cannot do that. I am well aware I have no driver’s license, no university degree, a job I got because I knew a guy, I was conned so badly I’m $3000 in debt, and I’m supposed to act confident in myself? I don’t see how I’m supposed to do that!

          • CatCube says:

            I:
            Everyone around me seems to treat being fiscally dependent at my age as normal. They have outright said it is normal. That’s what so confusing!

            That kind of is normal. I had financial dependence on my parents while I was going to college (they paid for a lot of my schooling, to the point I only had about $8,000 in student loans). It kinda starts to get unusual once you get to 26-27, but if you’re still in college it probably is likely. College sucks up a lot of money these days, where somebody really can’t put themselves through with a part-time job and full-time job in the summers.

    • Alex says:

      The main point of this reply is going to be that many things in your post are not neccessarily specific to your diagnosis but might happen to anybody.

      I am autistic (originally diagnosed with Aspergers Syndrome). I have gone No Contact with my parents because they told me that they would never consider me an adult no matter what I did,

      Many parents never consider their childeren adults no matter what they do, Aspergers or not. Of course this is not always admitted or talked about. I don’t want to talk down your conflict but to put things into perspective, being recognized as an adult by your own parents might not be a viable goal under the best of circumstances. Look at it this way: of all the people in the world you might convince that you are to be taken seriously, the people who have flashbacks of changing your diapers while you are trying to convince them are going to be the hardest target.

      while one family friend (D … K …) outright stated that I should give up on trying to be a full adult, and redefine “adult” based on my disability. My grandfather claimed that as an autistic I could never date or drive.

      First, if D … K … is your family friend’s name, why would you tell a name here in public? I’m not repeating it, so that it will be easier to edit it out of your post.

      Second, people give a lot of bullshit advice all day. So do I here, but you asked for it. Part of being an adult, and being recognized as such, is to get your own intuitions when to listen to advice and when to ignore it and move on. You do this by trial and error. I don’t know your grandfather, maybe he is just not a very nice person, but maybe he is well-intentioned and trying to protect you and prepare you for what he thinks is the reality of your future life. What I am saying is that such advice is mostly given implying that you will discount it for applicability to your own life on your end. The world is full of people who act as if they know exactly what is best for you. Most likely they don’t and you neither should take that personally nor expect to be able to change it. It must be hard when it comes from close family, but this is just how people are.

      -I mis-budgeted, … My parents had … student loan …
      But everyone keeps insisting this is normal, and puts pressure on me to spend more.

      putting pressure on me to accept free help

      I am trying to be financially responsible. This is impossible if I am living off loans and gifts, as people are constantly pressuring me to.

      but I don’t understand why people outright put pressure on me not to be fiscally responsible

      It is my understanding, that your parents are paying part of your expenses because you are a student. Correct? So, in some sense it is a “fact” that you are not fully an adult in that regard, since you are not fully taking care of yourself. And in some sense they are right, this is normal, very few 25 year olds really have their act together. So how would you be “fiscally responsible”? Stop being a stundent and start working full-time? Is that really an alternative? Maybe you just have to accept that in this phase of your life you need help from others (many students do, at least in my country) and that this is an aspect of adulthood you will not reach until you have finished your studies. But then the problem will go away.

      It seems to me that your problem is that “free help” is not actually free but comes at the price of creating a relationship of dependency and the helpers are making you feel that. This I think is also somewhat normal and something you will probably have to live with as long as you need their help, but not longer. Don’t forget to make a mental note to help them out one day.

      You repeat the pressue aspect a lot. If you can, try not to be pressured, try to be smart. If the help offered would actually help you, accept it! Even if it makes you feel odd and contradicts your larger goal of being “fiscally responsible”. [Footnote: Which I read as self-reliant anyways. There is nothing inherently irresponsible about accepting free help. On the contrary, it seems financially sound to accept it, if it’s free.] Try to value local improvements of your situation higher than grand principles with no immediate value, at least until you reach basic goals like making sufficient own money. Which is hard as a student. If, on the other hand, something really would not help, politely deny. Several times if you must. You might feel pressured but don’t let that guide your descisions. Don’t let the pressure out on the people who tried to help, but also please don’t hurt yourself. Professional help might do some good about this particular problem. Anyways, I predict that if you do this, people will see you taking responsibility and make your own descisions. Most people intuitively respect that.

      Out of excessive stress, I offered to restore contact with my parents IF … I sent terms of reconciliation to an intermediary (C.O.B), basically demanding my parents

      I don’t know you and I don’t want to sound condescenting but two things come to mind. a) it is very unlikely that a personal conflict will be solved by any sort of bargain. I can see why people would regard this as an emotionally immature thing to try. b) given that you want to be seen as an adult, you seem awfully interested in your parents’ opinion. There is this cultural ideal how family life is supposed to be and congratulations to the people who get to live that ideal. But sometimes it’s just not in the cards and maybe you should try to rely less on your parents and their opinions.

      • Dabbler says:

        EDIT: On your main point- You’re probably right, at least as far as I know. But, probably as an autistic, I hate the contradiction and my helplessness to do anything about it so much I don’t care. The sheer degree of consequences of being dependent on my parents for years to extremes most people never learn exist doesn’t help.

        ———————-

        On a general point: The reasons I care about being an adult so much are (a) This might sound very autistic of me, but I care a lot about the contradiction of being a nominal adult and still unable to be an adult in practice, and the fact that so many past generations have managed to become truly adult yet I cannot, (b) I have sacrificed so much time, money, friends (not counting my parents, five people have gone no contact with me over this dispute!), and time I could have spent enjoying myself (e.g. I even had to starve a day or two to pay for things, and spent two months without entertainment or socializing just trying to get my shit together. Tip of the iceberg), (c) Because I get discriminated against for being perceived as immature, and from my family and their connections it has been so much worse for being autistic.

        —————————–

        On my parents: You’re probably right on my chances, though it still bothers me deeply even knowing that because of the contradiction between being twenty-five, the age of adulthood, and my repeated failure to be an adult in any real sense.

        —————————–

        O.K, mentioning a name in public was stupid. I admit that. I would edit it out if I still could.

        You’re right on bullshit advice, of course. But it’s been the same bullshit advice from every single source, using very similar arguments. I am rejecting it from all sides, but I do want to try and figure out what is going on.

        Actually, though I am a student I also have a part-time job. I owe my parents and the university money, but I pay for my own expenses day-to-day. Apologies- I should have mentioned that.

        On relationship of dependency- Until such time as I screwed up my driving this should not have been necessary. As it is, it is only “necessary” in the sense that I need it until I can pay my debts, yet I am being pressured into something far more dependent.

        On your second last paragraph- (a) If I lose help I lose face with my parent’s friends, (b) It degrades my character to rely on others, and (c) I am. That is ignoring the consequences of dependency relationships, which have been taken to extremes in my past.

        I don’t have the kind of knowledge to know if you are right about people intuitively respecting my life path if I do it your way. But autistic though it may, it bothers me that in no objective sense could I be called “adult”. This is at about equal in importance to me as gaining the respect of others.

        On your last paragraph- I accept what you say about emotional immaturity. But I had (very rashly) said that I would take my parents back conditionally, and was obligated to create a reasonable set of conditions because I had given my word.

        I admit I have a greater-than-average need to be respected, because of the lack of respect involved. I desperately, desperately want to earn it. But it’s also true that I am trying to be an adult by an objective standard. I can’t just call myself an adult and make it so. Only if others start see me as an adult will I truly be able to be sure that I have succeeded.

        • Alex says:

          the contradiction of being a nominal adult and still unable to be an adult in practice, and the fact that so many past generations have managed to become truly adult yet I cannot

          From what you tell here you seem to be doing practically fine, the only problem being that you don’t get the confirmation you desire. Personally I think an important in becoming an adult is coming to terms with the fact that the universe has little explicit confirmation to offer you. So maybe spend less thought on how to get that confirmation and more thought on if you really need it and what for.

          Without knowing you it is nearly impossible to say anything helpful. You remind me of several friends – some of who could be considered to be on the spectrum I guess – in that you seem to live by rigid and explicit internal rules. I don’t know if you can see that such rules are important only to yourself. People in general will most likely not care, at least not in the long run.

          • Dabbler says:

            I know what you’re saying about confirmation. But confirmation from my parent’s peer group is linked up with several things- knowing I am seen as an adult is a kind of social status (of which I have precious little), it would mean an end to the social put-downs I hate (such as pressure to take loans), and it would provide psychological closure for childhood issues. So it’s not something insubstantial.

            There is also the point that I can be sure that I am not living the contradiction of being well above the age of adulthood but having failed to be an adult.

          • Alex says:

            There is also the point that I can be sure that I am not living the contradiction of being well above the age of adulthood but having failed to be an adult.

            I’m not aware of an defined failure mode of being an adult. The very concept is something I see discussed mostly by people who are not adults and maybe twens who use it self-mockingly. Think places with a very young core audience like e. g. 9gag.

            As for filling the idea with meaning. Death is an obvious candidate for a failure state. We can maybe extend this to not properly taking care of ones own childeren. Society expects you to make ends meet, within reason, which is something you say you are good at, were it not for that one incident. But even that is optional in most first world countries which have some sort of social security. And beyond that everything is largely arbitrary.

            So this whole question if you failed as an adult – it gets decided in your head and only in your head. I realize that it might be hard or impossible to you to change the way you think about these questions. However what you are looking for is a way to change what other people think about the same question. By analogy that should be even harder, shouldn’t it?

            Also such things tend to be self-fulfilling prophecies. Say by miracle you woke up tomorrow and were sure that you wern’t a failure as an adult. You would probably subtly change your behaviour. Other people would subconciously notice begin to see you more as an adult. The first step to convincing other people of anything is often to convince yourself.

            So you might ask how to convince yourself of something you don’t know is true. But like I said, not failing as an adult is a completely arbitrary category. There is no deeper truth to be explored. Maybe the person suggesting that you redefine adulthood also meant something along this lines.

    • bean says:

      Have you considered moving? I got a job in a city that I’d been to once before, and where I didn’t know anyone. In some ways, it’s been really nice, as nobody knew me before I was an adult, and a productive member of society. I’m not ashamed as to who I was earlier, and I wouldn’t mind going back to somewhere I’d lived before, but it is still nice not to have to deal with people who remember you being an actual child.
      The obvious downside is that you have to rebuild your social circle from scratch, which can be difficult, particularly if you’re autism spectrum. I’d suggest finding a good church, but you may not be religious. And of course, I don’t know if this is fiscally possible.
      Maybe even if you can’t move, it might be a good idea to look for new friends. Again, if you’re religious, find a new church. I’m not sure what the non-religious equivalent is, but find someone who hasn’t been ‘contaminated’. I will say that direct affirmations of doing well at adulthood are pretty rare, and in my experience they only come from the people who knew you before you were an adult.

      • Dabbler says:

        Thank you for the suggestion. If I had the money and wasn’t on a rental contract that will last for about eight more months I think that would be an excellent idea.

        EDIT: The rent could be theoretically transferred to a substitute if somebody wants to move in (if I remember correctly). The plane ticket and finding a new place… not so easy.
        (EDIT TWO: My mistake. Now I think about it, my work contract would be impossible if I left)

        • bean says:

          Could you use the 8 months to set up the move? Obviously, that’s a long wait, but it gives you enough time to make sure all your ducks are in a row.
          I’d also point out that moving an hour or so away could be 90% as effective as moving halfway across the country, and a lot cheaper.

          • Dabbler says:

            I’ve gone no contact with my parents and converse with everyone else over email.

            I think I will take your suggestion… once I have gotten my Arts degree. After about a year’s work for my boss plus a degree it shouldn’t be hard to get a lucrative job somewhere else. I’ll tell everyone socially that this is my plan when I next see them.

    • biblicalsausage says:

      Hi Dabbler,

      I don’t know much about autism. But here’s what I see just based on reading some of the things you’ve said. I might be wrong about some of this. And some of this might not be helpful. But because you’ve asked, here’s a few observations.

      Insisting that you are an adult, and that people have to say you’re an adult, looks childish. Refusing to speak to someone because they don’t think of you as an adult also looks childish. Offering to resume a relationship with your parents if they do a favor for you looks even more childish. You say that there are now five people you aren’t speaking to because of this. That sounds pretty extreme.

      I think there’s probably a lot more to this story that I do not understand.

      You mention that you are avoiding washing your clothes in order to save money. This sounds like it could become a serious problem for you socially, depending on how long you aren’t washing them. A huge problem socially.

      I am 26 years old and still finishing my bachelor’s degree. This is entirely my fault. I don’t expect people to consider me an adult yet. Some people don’t. This doesn’t bother me. You might feel better if you also didn’t expect people to consider you an adult while you’re still in school and still don’t have a driver’s license.

      You sound like you are obsessing over people considering you an adult to an unhealthy degree. It sounds like you’re willing to destroy multiple relationships over this.

      I hope things get better for you.

      • Matt M says:

        To expand on this a little bit, sometimes a thing that “real adults” must do is swallow their pride and accept help. If you are going in debt for driving lessons and people are offering to teach you to drive for free, it is childish to refuse their help because you want to prove how adult you are. While I understand a desire for independence, you should consider accepting the help that is offered to you (unless there are obvious strings attached)

        • Dabbler says:

          Suffice to say that there are.

          For about six years now, and arguably more, my parents have had a pattern which was supported by everyone else. Whenever I attempted a new skill or to become less dependent on my parents, I would face massive social pressure (such as intervention by my psychologist, my parents playing up the difficulties immensely etc) to give up. With no support whatsoever, facing a united front by others, and my own anxiety, I would give up- e.g. using the trains and trams by myself at 19, doing full time university at 22, dating at 23 etc.

          When I kept failing, people around me treated me like a child far more than they did to others around me- I assumed, only naturally, that this was a result of my failures. My parents kept blaming Aspergers Syndrome for my failures. Even when I succeeded by defying them they kept ignoring the successes and repeated the same pattern over and over again.

          They will take any excuse. When I fail a drive test a few of them claim I can’t drive (despite the fact my parents broke several promises that crippled me). I fail one relationship and my parents conclude I can’t date. I can’t afford to take any risks with my independence in this kind of company.

          So… yeah. There are strings attached.

      • Dabbler says:

        On insisting that I’m an adult: I’ve tried proving I’m an adult by accomplishing things. I don’t count my failure to get a degree because everyone around me ganged up on me socially until I gave up. Also, I repeatedly demanded to know what I was doing wrong to be treated as a child and was repeatedly told I was doing nothing wrong- it’s just that I had Aspergers syndrome.

        Empirical evidence by actually doing things my parents thought impossible didn’t work. Reasoning with my parents didn’t work. Asking what I was doing wrong didn’t work. And I had to do something.

        I went to a psychologist who said that I was subconsciously self-sabotaging because that was the only way to maintain my relationship with my parents. Even if I assume they were wrong, I know that parental pressure played a major role in destroying the only two relationships I had in my life (one because I agreed to my parent’s compromise before they broke my promise and the girl felt neglected, the other was traditional and very afraid when she found out my parents didn’t approve of me dating at all), that my parents had repeatedly and successfully lied to me about my capacity to accomplish tasks to sabotage me many many times (getting welfare, my first attempt at learning to drive when I gave up before even trying etc) and that despite my best efforts I had no way of stopping this.

        I see your point on washing. But right now I am desperate for money. I suppose I’ll have to soon anyway- meeting with my boss after a trip back is the kind of thing that requires a good wash. But it’s going to be hard to get the coins because the bank I withdraw from only gives cash in notes and the washer at my flat charges coins.

        —————————–

        I see your point on university and driver’s license. The point about it is that my parents and generation not considering me an adult is adding insult to injury. I was repeatedly pressured, including through the intervention of an autism specialist, NOT to go to university full time and pushed repeatedly not to go driving, AND I was repeatedly told that there was nothing wrong with this and it didn’t make me less of an adult.

        I keep trying to fix this about myself, and my parents and their friends keep getting angry at me and pressuring me NOT TO TRY AND FIX IT.

        • biblicalsausage says:

          Dabbler, I don’t know you, so I could be wrong here. But my guess is that all the people you know aren’t deliberately trying to sabotage you. They probably see you visibly struggling and want to help. And maybe the way they are trying to help isn’t always helpful, but I would bet that they’re trying to be helpful.

          Maybe they are right about some of what they are saying. The autism specialist thought you shouldn’t be in university full time, and now from the sound of it you are in university full time but you are having trouble figuring out how to keep your clothes clean.

          We all care about what others think of us. To some degree that’s a good thing. But it sounds like in your case this desire to be seen as self-sufficient could be causing pointless suffering for you. Maybe you simply aren’t in a position to be self-sufficient at this point in your life.

          If you are walking around in dirty clothes, that’s going to undermine your goal of having people think of you as self-sufficient much more than borrowing enough money for laundry would.

          If you want to fix things and improve your social standing, I would put wearing clean clothes above the goal of being financially independent from your parents.

          In modern Western cultures, self-sufficiency in dirty clothes isn’t seen as self-sufficiency at all.

          • Dabbler says:

            At first I thought they were just trying to be helpful too- my parents, and by extension my current helpers who all supported my parents. But so many of their arguments were obfuscations rather than things which would even sound coherent by SSC standards, so many things were said ignoring evidence, and so many times they irrationally stuck to a position in a way that blatantly defied logic (at one point we thought I was incapable of holding down a job- ever. So why did they insist I HAD to get a university degree for my own good?) that I lost all trust in it.

            It’s true I screwed up on driving. But though I didn’t budget well, not counting driving money I was losing so little in money that I had months to realize my budget was imbalanced before my savings were eaten. In a “what if” world where I had never tried to start driving, I wouldn’t have needed to borrow a cent, and could have paid for washing. I also get by day to day just fine.

            Nobody has ever said anything to me about self-sufficiency in dirty clothes being not self-sufficiency. Ever. They have said plenty about the need for self-sufficiency, before they started being hypocrites. If that was why they judged me I don’t see how I was supposed to know that.

            Just another reason to be angry at Mum and Dad, I suppose. They had me spend most of my school time with aides if not at home, claimed I would flourish when I got to university, I assumed they would teach me adult skills, and they never did.

          • biblicalsausage says:

            Most human beings have trouble saying things that sound coherent by SSC standards.

            It is unlikely that your parents, all your parents friends, a psychologist, and an autism specialist are all trying to ruin your life.

            Two other possibilities are more likely. Either they are saying things that are irrational by mistake, because people are often irrational, or else they are trying to say things that are rational, but they are having trouble effectively communicating what they mean to you.

            It sounds like you and they have very different ways of thinking and communicating. But the idea that they are deliberately trying to make you fail sounds a little bit paranoid.

          • Dabbler says:

            The psychologist and autism specialist were the same person. My parents both qualified in law degrees, one studied at Harvard, and the other became a skilled politician. Plus they are usually very coherent, except on one topic.

            I never thought they were deliberately trying to make me fail. But people can be ludicrously irrational and cripple lives. Victorian sexism and benevolent racism are very well documented, disability discrimination and helicopter parenting are well known. If they were in any way rational they would have meaningfully responded to evidence in ways they didn’t.

            I know what they were saying was not rational because of things such as refusing to let me drop out of university (back then I was such a manchild I didn’t dare go up against them) when they also thought I was never capable of getting a job ever, them repeatedly being proven wrong on things they claimed impossible, them not adjusting to being repeatedly proven wrong and changing their positions, and the fact that at times they used arguments that were blatant double-standards.

          • biblicalsausage says:

            Are you religious, Dabbler?

          • Dabbler says:

            I was Catholic. From age 18 on I didn’t believe the arguments my parents gave for it but didn’t dare say anything. I “officially” deconverted at around 23 and am now fully atheist.

            What’s your point?

          • biblicalsausage says:

            I was raised religious as well. I quite believing it when I was 21. But my parents still believe it all.

            My point is that almost all human beings are religious. And, I think you would agree, that their religious belief is irrational. But they keep believing it. And if I or you try to talk someone out of their religion, it probably won’t work. They’ll defend it, in very irrational ways.

            My point is that almost everyone has certain things they think that they cannot explain in a rational way. But when they give irrational answers to questions about religion, it doesn’t mean they’re being dishonest. A lot of religious people really do believe what they’re saying, and they really are trying to help non-believers like you and me.

            It’s just the way people are. It doesn’t mean they’re lying, or that they’re unusually bad people, or any of that.

            All of us have to deal with irrational people every day. Sometimes we’ll get angry at them. But obsessing over what irrational people think, or deciding to cut them all off from conversation only makes us miserable.

            We’re basically modified chimps. We’re social animals. We need other people, even other irrational people. We have to find a way to live with each other.

            And I’d bet me and you are just as irrational as our religious families. Just like they can’t see when they’re being irrational, there’s probably things me and you believe where we just can’t see that we’re being irrational.

          • Dabbler says:

            If you were saying my parents were persistently irrational rather than malicious, I think you’re sortof right. I just think that in this case their irrationality, like a lot of sexist prejudice, was so bad, and at the same time sufficiently avertable, that they should held account to their actions.

            My primary reason is that this culture has set a precedent that harmful sexism, when it reaches a certain degree of harm, is most definitely culpable no matter how much it is benevolent sexism. I believe that the logical conclusions of that as precedent lead inexorably to the claim I should blame my parents unless we introduce an ad hoc double standard.

            But even if we assume it wasn’t culpable, they were consistently finding new ways to sabotage me and my confidence. With the benefit of hindsight, I was a BETTER driver at this point then I was later on when I re-started. But I felt intimidated, I was psychologically beaten down, and I kept believing my parent’s promises (stupid I know!) and getting further confidence beat-downs and betrayals every time. So it’s not like I shouldn’t have gone no contact.

          • biblicalsausage says:

            Well, best of luck to you, Dabbler.

        • Barely matters says:

          Hey, from a guy who has been there:

          If normal washing is too expensive, as long as your clothes aren’t marked “Dryclean Only” or something like that, you can always hand wash them in the sink. Make sure your sink is clean first, use warm water. If you have some detergent or laundry soap around it only takes a few drops to make enough suds for a sinkful, if not, a little bit of shampoo will work in a pinch. Knead them with your hands until the water gets all grey and gross, then drain the sink, fill it with more warm water and repeat until the soap is gone and water is clear enough for your liking. If you can find some string you can run a drying line over your bathtub or shower to hang the clothes afterwords.

          It’s not quite as thorough as a machine wash, and I don’t know if it’s good for your clothes to do it all the time, but it’ll work, is almost free, and will go a long way towards looking put together.

    • WashedOut says:

      I’m not sure if this suggestion is appropriate given Autism, but here goes.

      It sounds like you are in need of getting out of your current environment for a fixed, short term and occupying yourself with simple, fulfilling work. Stop fretting in little circles around you trying to prove things to people, and start doing something that will mark the transition to adulthood.

      For example, Find someone to take over your lease and go and work on a farm or similar for 3 or 4 months. Your food and accommodation will be taken care of and you will earn a small wage. This should give you the necessary physical and emotional break from your family, and the labour will infuse your mind with a sense of purpose.

      Happy to elaborate more on why/how if this type of solution is applicable to you.

      • Dabbler says:

        That’s definitely an appealing idea. But it’s not practical. The only person I know with a farm is the very same boss who I owe mountains of work, and I’m told that there is far too little work to actually do there. And I’m not in a good position with anyone who might take over my lease.

        I can’t think of any autism-related reasons it’s a problem. Some autistics are more sensitive to noise or unfamiliar environments than I am so for all I know it would be for them, but not for me.

  20. Loquat says:

    After the recent article on polyamory, I’m now wondering – if we legalized poly marriage, what would that look like, and what changes might result?

    First of all, we’d have to nail down the basic question of whether we mean one marriage can have more than two spouses in it, or one person can have multiple active marriages simultaneously. Or maybe both! Each option would certainly have its supporters.

    As for the (American, since that’s where I live) legal benefits of marriage and ease of making the change…
    1. Hospital visitation rights: trivially easy to implement, no reason anyone would change this.
    2. Medical decision-making: people often bring this up, saying what if 2 spouses disagree, but that can happen already when 2 parents disagree on care for their child, so existing case law should work.
    3. Health insurance: I’m sure plenty of poly Americans who have good employer insurance would like to be able to put more than 1 partner on their plan. I’m also sure that most employers don’t want that, since health coverage is expensive and getting more so by the year. If poly marriage gets close to legalization, I’d expect corporate lobbying on this subject, and it might even help us get to a single-payer system, but might just end up with a law saying employers don’t have to cover more than 1 spouse.
    4. Inheritance when spouse dies intestate: will probably default to an even split between surviving spouses. Could get complicated if the deceased was a member of multiple overlapping group marriages, but not insoluble.
    5. Immigration rights: on the poly post Scott mentioned marrying the entire population of Mexico to circumvent immigration law – that’s easy to prevent by limiting a marriage to people who’ve actually signed on to that particular marriage, but there would very likely be attempts to abuse it like “these 12 Guatemalan men are in a totally legit group marriage with me, kindly give them all green cards so they can work in our family landscaping business”. Marriages of immigration convenience are already frowned upon by US authorities, though, so perhaps existing institutions would be able to keep things under control without legal changes.
    5. Child support in case of divorce: states that currently assume a married man is the father of all his wife’s children will have to change their laws if a woman can have more than one husband at once. Probably everyone will switch to requiring paternity tests. Or just saying every man a woman is married to is equally responsible for any kids she has, but that can get complicated fast if she’s changing the number of husbands between pregnancies. Also there’s the issue of whether non-biologically-related parents will be liable for child support, and able to sue for child visitation rights and/or custody in the event of divorce from both biological parents.
    6. Taxes: this is where things get really complicated, particularly if we’re going with the version where you can be in multiple separate marriages at once. My guess is that for simplicity’s sake Congress would just say all poly-married people have to use “married filing separately” status.

    So, who else has thoughts?

    • rlms says:

      Someone else’s thoughts are here (starting at section twelve).

    • Longtimelurker says:

      Marriage law is already a mess. Poly marriage would be much more of a hassle. On the bright side (if you have an awful sense of humor), the nasty divorces would be truly insane.

    • TheContinentalOp says:

      Is “Hospital Visitation” a real thing?

      Never once have I been asked to prove that I am related to someone (either friend or family) when I have visited them in a hospital. And if the hospital gatekeepers were to enforce spouse-only visitations, who carries their marriage certificate on their person?

      • Iain says:

        It’s a thing (or, rather, it was until relatively recently).

      • Gobbobobble says:

        I could be mistaken, but I always understood it to be A Thing for “visitation outside the Official Visiting Hours” and visiting during Official Visiting Hours is generally wide open.

  21. Odovacer says:

    Why/how did nerd glasses become popular? I’m referring to the thick framed, larger lenses glasses. When I was in high school and early college, thin framed*, rectangular glasses and smaller lenses were more in vogue. Large glasses were for cops or old people.

    I’m in grad school and the vast majority of undergrad who wear glasses have the thick-framed larger ones. I wonder when smaller glasses will come back into vogue.

    *Look under short faces for an example.

    • Wrong Species says:

      Nerds don’t actually look good in nerd glasses. By handicapping themselves, attractive people signal that they are still attractive.

    • hoghoghoghoghog says:

      Counterquestion: do clothing styles ever happen for an identifiable reason? Honest question, not snark; I’m no social historian and I don’t know how granular they can get.

      • LHN says:

        I think some can be traced back to particular celebrities popularizing them– though a lot of the most famous examples (like Gable in “It Happened One Night” tanking undershirt sales or Kennedy killing the hat for men) turn out not to be well supported. And even if so, that kicks it back to the question why some celebrity fashion choices catch fire while others remain isolated idiosyncrasies.

  22. In my city of Minneapolis, it is almost certain that a $15 minimum wage law will be passed this year. I think this is a tragedy for the very low skilled in the city, who may never work again in their lives. At least it will just be in the city, so the low skilled may be able to find jobs in the suburbs, however, those with few resources may also find it difficult to commute that far.

    I wrote a posting about why this is bad, but it is kind of long to put here, so you can find the posting here.

    A summary of my arguments:
    1) The money has to come from somewhere, and consumers don’t have any more money than before. Thus they will likely spend less on those areas that employ minimum wages workers, such as eating out. These industries will shrink and unemployment will result.
    2) Those making the $15 will mitigate this to some extent, because they have more money. But it only mitigates to the extent these $15 wage earners spend money on the minimum wage industries, which is likely not very high.
    3) Raising minimum wages so much gives a lot more power to employers, because they will have their pick of workers at the new higher wage. So the $15 wage earners will have more money, but their jobs will be tougher.

    Please let me know of any holes in my argument.

    • skef says:

      Your #1 ignores the fact that virtually all product categories, including, for example, restaurants, are available in different levels of quality and price. And those levels don’t always correlate neatly with employee salary levels. Your “Eat out/stay home” dichotomy either assumes there is one level, or only pertains to people who only eat at restaurants of the lowest level, many of whom will be in the class that benefits from the higher minimum wage. Others might slide down a tier. The overall effect is … difficult to predict without a model.

      “Minimum wage industries” is a huge oversimplification, as most businesses employ workers with different wages. (Your write-up, as opposed to this summary, only relies on that to some extent. You say in the former that non-fast food restaurants have higher labor costs, and argue that they will therefore wind up with a higher price increase, but base that on no figures about minimum wage level labor.)

      The “no one knows what will happen, but here are the intuitions I have about why it’s so ‘risky'” numbered section construction amounts to sophistry. As you note, people study this stuff and build models. Perhaps they are flawed, but what are the chances your pile of intuitions is less flawed?

      The moral intuitions you use in that section are grounded in theoretical commitments you don’t outline or defend. Suppose that a BSE outbreak doubles the price of beef, and as a result “the $8 per hour worker loses his job and … never finds a job again in his life?” Does that call for beef price controls, or is that just life? One difference, I suppose you might argue, is the omitted phrase “because he isn’t worth $15”. Economies are dynamic systems, and this is a weird and unjustified reification.

      This section is also incoherent. The extent to which “the raise to $15 is overstated” works against the rest of your argument. And “flexibility in working hours”! Are you familiar with the ongoing controversy over dynamic scheduling? You do know that the benefits of “flexible scheduling” in lower-wage employment now accrue almost entirely to employers, right?

      Finally, almost all of your arguments work equally well against present minimum wage laws. You don’t argue that they should be eliminated, or if not why not. The combination is rhetorically dodgy.

      • Thanks skef for your comments. I did ask for holes in my logic, so I won’t complain about you trashing my essay. But I think your critique is wrong in most cases.

        our #1 ignores the fact that virtually all product categories, including, for example, restaurants, are available in different levels of quality and price.

        This one is partly right, however, which I didn’t consider. Generally the higher tiers don’t employ many low wage workers, so those won’t be affected, but there may well be a tier or two with such workers. This may mitigate the affect I discuss a bit.

        “Minimum wage industries” is a huge oversimplification, as most businesses employ workers with different wages.

        Well yes of course, I wasn’t trying to be quantitatively precise. There have been lots of studies on minimum wages, and I don’t know that any of them have been very useful because they are for too short a period to properly determine the effects. These studies imply much more precision than exists. But it is true that the average person will be more impressed by a heavily quantified study over my discussion, even if the study is pure BS.

        Perhaps they are flawed, but what are the chances your pile of intuitions is less flawed?

        Pile of intuitions? What are you referring to? That consumers will buy less when prices go up? That firms will then use fewer employees? Those are firm economic theories; I didn’t simply intuit them? What do you mean by this?

        Economies are dynamic systems, and this is a weird and unjustified reification.

        This whole paragraph makes no sense. Without the minimum wage increase, the $8 per hour beef workers finds a different $8 per hour job if his industry has a crisis. That is much less possible in the $15 minimum wage scenario, BECAUSE the economy is less flexible.

        This section is also incoherent.

        Which section do you refer to? Perhaps you are right that flexibility in hours is more difficulty than a feature to most low paid workers. I think about my kids’ jobs, which have been in retail and restaurants. My kids could state days or times they couldn’t work, which is a flexible benefit, but some employers have been pretty abusive about last minute scheduling of inconvenient hours.

        Finally, almost all of your arguments work equally well against present minimum wage laws.

        Not equally well at all, because there are a whole lot more workers that are currently paid between today’s minimum wage and $15 than those that are currently cut off from working because of current minimum wages. IT is true that minimum wages today cause some very low skill workers to be unemployed for life, but this change will dramatically increase these numbers. If I try to talk about every related topic, my essay gets too long and the main message is lost. How is that rhetorically dodgy?

        • skef says:

          Your pile of intuitions are your numbered claims. Specialists build models to try to determine the impact of economic changes. You talk about the opinions of economists on surveys, but in relation to research you say:

          No one knows exactly how much unemployment will increase with a $15 minimum wage, especially considering unknown confounding events such as general inflation and the strength of the economy, but it is clear that it would be a very risky step.

          Then you proceed to explain how risky in terms of a collection of observations divorced from any model: “No one, including specialists, can really know … but I have a pretty good idea.”

          This whole paragraph makes no sense. Without the minimum wage increase, the $8 per hour beef workers finds a different $8 per hour job if his industry has a crisis. That is much less possible in the $15 minimum wage scenario, BECAUSE the economy is less flexible.

          The point about a rise in beef prices wasn’t in relation to the beef industry, it’s in relation to restaurant prices.

          Restaurant prices can go up because of increased labor costs or because of increased ingredient costs. A rise in beef prices would therefore have the effects on much of the fast-food industry that you envision in your examples, given your presumption. Under those conditions I doubt you would call for price controls. More narrowly:

          Without the minimum wage increase, the $8 per hour beef workers finds a different $8 per hour job if his industry has a crisis.

          I thought your reification was just unwarranted, but this is nuts. You are claiming that individual employees (presumably on the low end) tend to have a fixed dollar per hour worth that is otherwise fungible. Move employees from the dishwasher to the factory, this person is $8/hr, this person is $10/hr. Who thinks this way? Employers don’t. “Unskilled labor” is a euphemism that mostly has to do with non-experience-based qualifications; it doesn’t mean un-differentiated.

          The incoherent part is #5 (which I refer to in the next sentence). Your overall argument is not to revoke the minimum wage, but that $15 is too high. Your argument in #5 is that setting a legal amount of $15 will result in a nominal amount of somewhat less than $15. Given your other arguments, that’s good, right?

          Not equally well at all, because there are a whole lot more workers that are currently paid between today’s minimum wage and $15 than those that are currently cut off from working because of current minimum wages.

          Supporters of higher minimum wages often argue that increases will have no effect on unemployment. There are reasons to think that’s probably wrong. Changes in levels of unemployment open up rights-based arguments based on depriving those who will become unemployed, which you include in your write-up.

          Now you’re going further and making assertions about the degree of employment increase. A model could try to predict that, but you have no reasonable basis for prediction. Other things you’ve said imply that your thinking is: “All these $10-$15/hr people will be out of a job.” This is lunacy; economic effects are never that simple.

          • skef says:

            On the last point, here is an article on a minimum wage increase in Seattle by someone who broadly agrees with you. Note that the claimed effect is relatively modest.

          • Ah it is my contentions at the bottom with which you disagree. Yes. those comments are more speculative than the rest of my essay. In the first part of my essay, I show that raising the minimum wage to $15 will have a detrimental effect on low skill workers, but don’t really talk about how detrimental. The last part was to respond to those who claim the correct level of a minimum wage depends on the elasticity of labor demand, because any given minimum wage might result in higher aggregate wages of low skill workers. I don’t think anyone truly has a good enough grasp of this elasticity, so it is hard to get grip on the argument.

            So my response is a laundry list of objections as to why the $15 minimum wage is a bad idea regardless of any mathematics that may arise. #4 through #6 were to show that the benefits aren’t as high as the proponents indicate, so these should be depreciated in any cost/benefit analysis. #1 through #3 talk about the costs being higher than generally calculated. #2 is totally normative; it isn’t fact based at all. Do you disagree with the rest? I agree that the flexibility of hours is maybe wrong, but otherwise you haven’t disagreed with any of them in substance.

            I am still totally confused by your example of the $8 per hour worker whose industry takes a downfall. I don’t see how this contradicts anything I have said; if anything it reinforces my argument because this guy will likely never find a job at $15 but probably will get another job if the wages are free to fluctuate with the market.

            I have never implied that all the $10-15 per hour workers will be out of a job. My argument is that a large portion of the $8 – $15 workers will be out of a job. My guess it will be significant minority.

            I read that link by Worstall before. I have seen smart comments by Worstall, but this isn’t one of them. He wrote this essay a year ago, only a few months after small employers were required to pay $12 per hour. This is far too soon to see much of any effects of a $15 minimum wage.

          • skef says:

            We seem to just be talking past each other at this point.

    • IrishDude says:

      The argument against minimum wage that speaks most to me is that two consenting adults should have the right to make agreements without coercion from others. My body, my choice of what compensation I’m willing to take for my labor.

      Not sure I agree with your 3), though. Changing the minimum wage doesn’t change the pool of people who are profitable to employ at $15/hr, and if the pool of qualified applicants/employees doesn’t change then I don’t see much pressure on that pool of people to take on more responsibilities (is that what you mean by tougher?) for the same wage they were able to be employed at before the minimum wage increase.

      • Matt M says:

        Changing the minimum wage doesn’t change the pool of people who are profitable to employ at $15/hr

        I think it could, if you make easier/more desirable labor more competitive with more difficult/annoying labor.

        Perhaps today a fast food fry cook makes $8/hr, a somewhat skilled (associates degree or 1-2 years of experience) assistant bookkeeping clerk makes $15/hr, and an unskilled garbageman also makes $15 an hour.

        If you suddenly decree that fry cooks must make $15 an hour, the bookkeeper and the garbageman are no longer properly incentivized to obtain higher-value skills or to work a more unpleasant job. The wages of bookkeepers and garbagemen may rise in an attempt to retain workers, but economic concerns may prevent them from rising at the same rate or to the same extent. So at the margin, the somewhat bright HS graduate may forego the associates degree in accounting, because they can do almost as well as a fry cook. The garbageman may rather work in a restaurant than handling stinky garbage all day. This would increase the pool of people competing for fry cook jobs.

      • Paul Zrimsek says:

        While in the longer run employers will adapt by substituting away from low-skilled labor, right now their ability to get along without it is limited– and paying $16 for someone who’s actually worth $15 is likely a better deal than paying $15 for someone’s who’s worth $10.

        • Trofim_Lysenko says:

          Since a lot of minimum wage businesses are low-margin and staff for things like customer service, an easy fix is simply to cut staffing to minimum levels.

          If I need a peak of 5 employees at Burger World for the Lunch and Dinner rushes to consistently hit 90-120 second Order-to-Drive-Off times, maybe I cut to three employees and simply soak up the decreased customer satisfaction from slower service, for example.

          Fast Food restaurants and registers in stores can function at lower staffing levels, it just translates directly into customer wait times going up. If you were 24 hours, cut your grave shift, etc.

          In theory, a hold-out store staying at full staffing could steal business away by touting their superior customer service in this environment, but I wouldn’t expect that to happen except in situations where customer service was the absolute lynchpin of their business model (as opposed to quality of product).

      • Not sure I agree with your 3), though. Changing the minimum wage doesn’t change the pool of people who are profitable to employ at $15/hr, and if the pool of qualified applicants/employees doesn’t change

        Say there are 1000 people in a given town earning less than $15 per hour. With an increase in the minimum wage to $15, then it is only profitable to employ 700 of these workers. Obviously the employers will keep the 700 most productive workers, but that doesn’t mean that the other 300 wouldn’t be able to do the same jobs at the firms, and the firms still make a profit.

        There are undoubtedly a few workers that wouldn’t be worth paying $15 even if no one else was available, but in my experience there aren’t many that are that low skill. The value of any worker is based on supply and demand. Theoretically if we killed off half the low skill workers, the remaining ones would make a lot more money because of the decrease in supply.

        Back to my example. There are 300 potential workers out there, most of whom could do the same job as the current workers, even if slightly less productive. The employers will be under more pressure, since they are trying to survive in a tougher cost environment. So even the more ethical employers will push the workers to be more productive, and use the threat of the 300 available workers out there as the incentive. The nasty bosses will find it easy to abuse the workers, since none of them want to be part of the 300.

    • The Nybbler says:

      Eventually inflation will erase the effects of the new minimum wage, so, depending the economy, “never work again” may be an overstatement.

      • Matt M says:

        At which point the left will demand the minimum wage double again in order to compensate for inflation.

    • James Miller says:

      Flex schedules are now a big part of (3). You tell employees: “I will tell your the night before when you will be working, make sure you are free to work anytime although I won’t actually guarantee you any work.”

      • Trofim_Lysenko says:

        What minimum wage business functions along these lines, though? Most of the ones I can think of (retail, low end food service, call centers, hotel front desk, etc) require pretty fixed numbers of employees for minimum skeleton staffing.

        For example my subordinates make less than a dollar over minimum wage (the local burger king is hiring for a quarter more an hour in fact), but I couldn’t make my department work on that sort of schedule. I can cut people, but even cutting things to the absolute bone I’d need at least one person at the desk from opening time to closing time, two if I don’t want to close the desk for the FLSA-mandated 30 minute break.

        • James Miller says:

          Prediction: you live in a big city where the market wage for unskilled labor isn’t too much above the minimum wage.

        • Trofim_Lysenko says:

          I live in a rural town of 35,000 2-3 hours drive from any large cities (large defined as 100K plus).

        • Trofim_Lysenko says:

          Not intended as a gotcha, I’m genuinely curious: What minimum wage/unskilled labor businesses can operate on a flex staffing basis long-term?

          I can think of some piecework stuff that works that way (agricultural labor can be highly variable), but nothing hourly.

          • Matt M says:

            Why not? Anecdotally, I’ve heard various friends and family members in service-industry jobs (cashier, fast food, etc.) complain that their schedules were highly variable, their hours not guaranteed, that they could be called in with little advance notice and fired if they don’t show, etc.

          • Trofim_Lysenko says:

            Because any service industry business has a man-hours floor. I sat down and worked it out for my current job and if we just didn’t care at ALL about customer service, we could do it with two supervisors, one part-time “shift lead” (to cover the supervisors’ days off), two full-time hourly personnel, and 1-2 (probably 2) part-time personnel.

            So, even at absolute minimum, that’s two hourly people who will not only be full time, but have a pretty fixed schedule. Even the 2-3 part timers would be pretty much guaranteed at least 16 hours a week. But customer service DOES matter, so we need more than that absolute minimum number of employees. Plus, you need some padding to cover things like PTO (if your business offers any), people calling in sick, and just plain unforeseen crises. Customer volume is variable but it’s not THAT variable: If you’re a Taco Bell, you know you have a lunch rush, a dinner rush, and depending on your location and hours maybe a 1-2AM rush when the bars close on Friday and Saturday nights. You start with that core of full time people and supervisors, and then you bulk up your staffing so that the peak people are there during the peak times. And here’s where the “hours not guaranteed” part comes in: As much as possible, you do that padding with part-time employees (but they can’t ALL be part-time or your costs and churn will be too high in the long run, I can explain this if you find it hard to believe. Short version is that training and onboarding people, even at the lowest levels, has associated costs).

            That gets you the sort of complaint about not getting guaranteed hours you mentioned because you staff to ensure you have plenty of people and then you cut people early, push them back, or call them off entirely if it’s slower than anticipated. That still leaves a massive gap between “I’m part time but some weeks I only get 15-20 hours instead of 25” and James Miller’s “I’ll call everyone the night before to tell them if they’re working or not”. That’s simply an insane way to run a business with fixed hours. Customer volume is variable but it ain’t THAT variable, and it’s a pain in the ass trying to get part-time hourly employees to come in on short notice for legitimate unplanned contingencies like someone calling out on you, much less trying to keep them strung along like that.

            So what you end up with is a staff that has a small core of full-time people who comprise the bare minimum you need to keep your place running, and then as much as possible you fill the rest with part-time people, but not ENTIRELY with part-timers. If you get someone who’s actually a dependable and reliable team member it’s generally worth it to make them full-time unless you know they’re not sticking around (e.g. Student). For example, at my work we actually have 3 supervisors, not 2, supervising 10 hourly team members, of which 4 are part-time. Some teams you’ll see a much higher number of full-time employees because you have a much more steady level of labor demand (e.g. Overnight Stockers at Wal-Mart, a job I’ve also worked. You know how many trucks you’re getting in and when, generally speaking, though you can get surprised, so it tends to be more full-timers than part-timers). Some, you’ll see something more like “skeleton crew of full timers, everyone else is part time”, but again even there you’re going to have full time employees and even the part timers will have relatively fixed schedules coming from the management side.

            Past that, it all comes back to the other employees, with the big disruptions and “come in on short notice” issues due to the fact that part time (and often even full-time) low hourly rate employees tend to be unreliable, for a variety of reasons, which is a whole post in and of itself. If they’re reliable and good workers, they don’t stay at that level long and within 1-2 years leave that market segment for better positions unless there is some external factor in their life.

            Now, some businesses will try to hire ONE or maybe two employees out of a 15-20 person team and keep them on that sort of “on call” basis, as a way to cover people who call out sick. But in my experience (coming up on 5 years here as both hourly and more recently salaried, 3 years at Wal-Mart hourly, various other retail/service positions in High School) unless you actually find someone who WANTS to be on that basis, they will bail on you within the first few months.

            So if it’s all but impossible to keep 1-2 employees out of an entire team around on that basis, I find the claim that this has become a common arrangement for a lot of workers very unbelievable.

  23. Controls Freak says:

    Working for the federal government, I get to sit through wonderful things like annual sexual harassment training. Its only bearable because of some of the utterly ridiculous/hilarious things that they say. Highlights of what I learned this morning:

    1. Inappropriate sexual words include such horrors as “girl” and “boy”. I’m dreading the day when a coworker happily informs me of a pregnancy. How will we ever dance around the naughty words?

    2. They had a spectrum of behaviors for sexual harassment, with “opportunities for intervention” at each stage. Problem is that the lower two cases explicitly called out the behavior as consenting (one was consensual flirting, and I don’t remember exactly what the other was, but it specified consensual). I get that there are good reasons to want to minimize office romances in general, but that is not “on the spectrum of sexual harassment”.

    3. “Sexual harassment is a form of gender discrimination.” Really. At this point, “discrimination” has no meaning besides “is bad” (unless it’s Good People like universities price discriminating in favor of other Good People).

    4. “It’s not sex when she’s wasted/unconscious/etc.” No. You don’t get to just redefine the word “sex”. It’s bad sex that is really immoral, and we’ll probably throw you in jail for it… but it is sex.

    • One of the best things about being a contract worker is that Human Resources leaves me alone.

      It’s too bad that such “training” is so terrible. Sexual harassment is a real thing, and with training like yours no one takes it seriously. I wonder if the teachers of this training realize that their training may well result in less concern about harassment instead of more. Of course the organizations themselves don’t care at all. They are just shielding themselves from liability. When they get sued because someone was sexually harassed, they will trot out the training they gave to show that the organization wasn’t at fault. The fact that the training was so terrible doesn’t matter at all in the courtroom.

    • CatCube says:

      That’s stupid even by the standards of the Army. What agency is this? (If you don’t mind answering)

    • Eric Rall says:

      3 is actually how sexual harassment became recognized as an actionable tort: there weren’t laws specifically against sexual harassment, but there were (and still are) laws against gender discrimination which provided for civil remedies.

      The legal theory which was successfully argued and accepted by the courts is that harassment is a mechanism for gender discrimination. And in the contexts in which this argument was being made (cases where female employees were being pressured to have sex with their male supervisors or else risk getting fired (quid pro quo harassment), and cases where harassment was overt and pervasive enough as to be a real barrier to women functioning as employees at the firm (hostile work environment harassment)), this was a plausible argument.

      I think the disconnect you’re seeing is that for the most part, the once-central examples of sexual harassment have become relative rare and mostly concentrated at poorly-run firms (where there’s a cultural dysfunction as well as a lack of regard for legal liability). And when they do happen at relatively well-run firms, the perpetrators are generally careful to hide their actions from everyone but their victims. So you’re looking at noncentral examples that are being swept up into the same bucket by an institutional culture trying to keep central examples of harassment from happening (and from being blamed on the agency rather than just the individual when they do happen), similar to the Talmudic concept of “building a wall around the Torah”. So things like 1 and 2 are the Sexual Harassment equivalent of taking a Torah prohibition against boiling a calf in its mother’s milk and turning it into a rule where you can’t eat meat and dairy in the same meal and you have to have separate sets of dishes so you don’t accidentally get a stray cheese molecule from a previous meal on your lamb chop.

    • Longtimelurker says:

      At the risk of being overly dismissive, relax. The as a person who just had mine on Tuesday, it is just standard (swear word meaning ones gluteus)-covering. It is entirely to give the appearance of being helpful, rather than the substance. But, yes, HR is the scum of the earth.

      • herbert herberson says:

        Yeah, that comment doesn’t exactly convince me that sexual harassment training is unnecessary and duplicitous

    • Nabil ad Dajjal says:

      Something interesting I’d like to see would be sexual harassment training that wasn’t “abstinence only” so to speak.

      The way it has been explained to me both at my present position and in undergrad, there’s no way to date a co-worker which doesn’t constitute sexual harassment at some point. So when co-workers inevitably date against the rules the guy is in a very vulnerable position.

      Now some probably consider that a feature rather than a bug. It might be too much risk for too little benefit for employers to give good advice. But it would be nice.

      • Deiseach says:

        The way it has been explained to me both at my present position and in undergrad, there’s no way to date a co-worker which doesn’t constitute sexual harassment at some point. So when co-workers inevitably date against the rules the guy is in a very vulnerable position.

        From the employers’ point of view, it’s a disaster waiting to happen. The best outcome is Jack and Jill fall in love and decide to get married, which at least makes it no longer your problem. But if the love affair goes sour, you probably have two people working in proximity who now hate each other’s guts (or one of them spends every spare moment crying in the bathroom because they just saw their ex) and it not alone affects their work, it affects the atmosphere of the workplace and their co-workers. Maybe there’s even the chance of a lawsuit, if one party was in a subordinate position or otherwise feels sufficiently aggrieved and hard done by that they are accusing the organisation of not protecting them.

        Think of that Uber story where a manager allegedly was hitting on women working under him on the excuse that he was in an open relationship and he couldn’t help himself, the women were just too tempting.

        Or a case from my last place of employment, where a middle manager left for a better position in another town. After he’d gone, it turned out he’d been having an affair with a subordinate (luckily, in another department, so not under his direct command) and she was now pregnant. With twins. And he was married. But had left his wife and taken up with a new person in the town he was now living and working in.

        I don’t know if she could have taken out a sexual harassment case, but seeing as how he was in a superior position to her, maybe? It would have been very sticky for the organisation, anyhow!

        So how do you intervene in a case like that, if you have an inkling A and B are getting closer than simply a working relationship, and A and/or B are married? Calling them in and telling them to knock it off runs the risk of “how dare you, we’re not doing anything of the sort, this is interference in our private lives” and a complaint being lodged against you. Do nothing, and A or B may then later on lodge a sexual harassment complaint, especially if one party was senior to, or the boss of, the other, including “you should have known what was going on and it was your duty to put a stop to it”.

      • Matt M says:

        Something interesting I’d like to see would be sexual harassment training that wasn’t “abstinence only” so to speak.

        An interesting analogy. Somewhat amusing that abstinence only is mocked and ridiculed in the concept of sex-ed (lol yeah right, teenagers are going to have sex no matter what so we might as well show them how to do it safely) but somehow, once you graduate high school, it is assumed that we can just order you to never have romantic feelings for the people in your own age group with similar interests that you’re locked in close proximity with for 8-12 hours a day and that’ll be just fine and anyone who breaks the rules is just an awful person and it’s entirely their own fault and who cares if they suffer for it?

        I’m now also imagining a young woman (or her parents) suing a school for offering sex-ed, under the guise that they promoted and condoned behavior that would logically lead to teen pregnancy.

        • Aapje says:

          @Matt M

          The difference may be that the religious abstinence stuff tends to include and perhaps focus more on women, while the sex ed abstinence stuff pretty clearly targets men.

        • johnjohn says:

          Is it really all that amusing that we treat teenagers different than adults?….

          • Matt M says:

            If by “teenager” you mean 17 year old and “adult” you mean 20, then yes.

          • johnjohn says:

            I mean, most of western society treat 17 year olds and 20 (21, if it’s the states) year olds pretty differently.

            Also the vast majority of the workforce is not 20 year olds

    • Deiseach says:

      “Sexual harassment is a form of gender discrimination.”

      Well, that’s going to get them in trouble, because it implies that the only sexual harassment that can happen is between persons of opposite genders where one person is female-identified and is the victim of the harassment. What about male-identified persons being harassed by female-identified colleagues or superiors? Or persons of the same gender?

      Inappropriate sexual words include such horrors as “girl” and “boy”. I’m dreading the day when a coworker happily informs me of a pregnancy. How will we ever dance around the naughty words?

      We’ll have to go back to Victorian etiquette and be suitably discreet and euphemistic in how we refer to happy events. Do not presume the gender of the conceptus, after all before ultrasounds no-one had an idea of the biological sex until birth. Restrict yourself to expressing your best wishes for the safe delivery of a healthy child, though “healthy” may also get you in trouble because it is ableist and expresses the preference for a neurotypical, able-bodied child. Let the party in question raise the subject of assigned/assumed gender 🙂

      I sympathise; currently my place of work is in the throes of implementing an entirely new government framework of codes of practice and we’re in the midst of ploughing our way through showing how we are engaging with “Principles, Standards, Components of Quality and Signposts for Reflection” via 16 different Standards of Quality, further broken down into sub-sections, where we have to provide evidence for every point as to how we are in compliance. And we’re going to have a visit soon from the relevant officer to check up on this, and their report will affect our funding.

      This on top of the other standards from other bodies we have to abide by, as well as getting everything ready for the annual audit, and coping with the usual crises of day-to-day running of the service 🙂

    • Machina ex Deus says:

      Yeah, Federal agency sexual-harassment training does suck: a lot of bad definitions and unrealistic situations, presented to seated people with no class interaction.

      What I’d really like to see is more hands-on training.

  24. sierraescape says:

    Hello. My aunt leads a fairly popular (~20-100 shares per post, 20K likes in total) anti-vaccination page on Facebook. I know her pretty well, and she seems like an intelligent person who has gotten fixated on this idea and other associated ones. How can I build a good case for vaccines? Keep in mind she may not believe statistics regarding their efficacy.

    She links sites like this which just contain too many statistics to properly refute all of them (if she is even able to trust the refutations) so I’m really looking for some kind of knockdown argument if one exists.

    • LHN says:

      No good ideas, but I hope you find some. I just found out a teenaged cousin of mine who’s already had to deal with way too many health problems has pertussis. (Despite having been vaccinated himself.)

    • smocc says:

      – Rule number 1 is do not use Facebook. It is the worst possible venue for changing minds. Similarly, I recommend focusing on your aunt only, and not her group. Being told you’re wrong by a stranger is basically the worst, and people do not usually respond kindly (just see what happened to old Isaiah)

      – Rule number 2 is that logos rarely persuades. Before a set of facts can convince someone to change their mind, they have to have decided to allow facts to change their mind. This is not the default state of mind for most people. Anti-Vaxxers seem to largely have decided (perhaps unconsciously) to reject a large set of authoritative sources. Your initial goal should not be to present these facts, but to help your aunt reach the point where she is willing to consider her choice of beliefs critically. Remember, many apparently beliefs about facts are actually about something else, even for knowledgeable people.

      Considering rule 2, your job actually comes in roughly three stages:
      1. Act as a model of someone who shares her fundamental values and basic knowledge, but still believes in vaccines.
      2. Provide an initial opportunity to question her beliefs about vaccines critically, maybe by challenging them directly, or maybe by asking critical questions.
      3. Act as a guide in re-evaluating facts and forming new beliefs about vaccines. Point out better sources of facts, and provide answers to questions.

      Stage 3 is the most fun, but stages 1 and 2 are the most important, and real belief change rarely comes without them.

      – Re stage 1: Consider why you want to change her mind. Concern for her children? Concern for society at large? Simple dedication to people not being wrong about things? There are no dealbreaker answers here, but be honest about whatever your reason is. Do not pretend to have a motivation that you do not have, as people can sense this and shut down against insincerity.

      – Consider your relationship with your aunt. Are you close to her? Do you like her? Does she like you? Does she think that you like her? Again, no dealbreakers — except for “she hates me and thinks I’m a judgmental know-it-all” — but it affects what kind of approaches you can take and which will likely fail. For example, if you are not close, may want to work on convincing someone else close to her first. You might not be the person to accomplish all three of the stages above. (Paul plants, Apollos waters)

      – While you should be honest about your motivations, you should also be open to learning from her. She might not know anything about medical science, but you might be able to learn why anti-vaccinism is so important to some people. (I myself am curious, I just don’t get it, and I don’t have any close friends I can ask.)

      – Re: stage 1, anti-vax beliefs are not likely fundamental to your aunt’s identity. Rather, they are more likely the result of more deeply held beliefs and desires: to be a good mother, to be self-reliant, to think for herself. Thankfully, she does not have to change her mind about any of these in order to be persuaded about vaccines. In fact, vaccine use is more compatible with all of these than not! (But careful, do not assume these are actually her fundamental values. They are a good starting guess, but ask questions when appropriate.)

      – Re: stage 2, when possible, lead with questions, but don’t hide the ball. For example, “I’ve noticed on Facebook that you are anti-vaccine. Would you mind talking with me about that? The anti-vaccine movement has never really made sense to me, and it even seems a little dangerous. For example, [why do you think vaccines cause autism? / why do you think the medical establishment is wrong about this? / why aren’t you concerned about a resurgence of measles etc.? / why don’t you trust such and such source? / what sort of sources would you trust? etc].” By asking questions you hopefully allow her to consider her belies critically without taking on a defensive mental stance.

      – Re: stage 3. Stage 3 is usually the most straightforward, but you do need to know your stuff. If you can’t answer reasonable questions, or at least know where to look for the answers, this will be very difficult and you may undo progress you made in other areas.

      Finally, don’t expect to be able to change anyone’s mind in a single conversation. In my experience, most belief change happens after conversation, during private reflection. This can feel frustrating at first, but it’s actually nice because it means you can have influence even when you initially think you have failed.

      (Take this all for what it’s worth. My experience persuading people was as a missionary and I have no idea if it generalizes well. I have never persuaded an anti-vaxxer before. I wish you well in your attempts, and I think it is a good idea to try.)

    • keranih says:

      @ smocc says very wise things here.

      There are upsides and downsides to everything, so I don’t think that you’re going to find a knockdown that works on someone who is aware (correctly) that vaccines have some downsides.

      So, if it was me, and assuming I couldn’t leave it alone (I have a higher tolerance for friends and family being wrong than than strangers) I wouldn’t start trying to convince her that vaccines were 100% right all the time. I’d allow that sometimes they are problematic.

      Instead, I’d go to the other end, and ask her to consider when *a* vaccine (not as a group, but a particular one) would be a good idea. Like, sure, everyone should eat healthy, sleep enough, etc etc. What about for the marginal populations in a city in west Africa facing an Ebola outbreak? Would that be a good time for a one-off use of vaccines? What if they ran titers on everyone, and only vaccinated the people with low/no titer?

      Once *you* agree that vaccines can have issues, and *she* agrees that sometimes they are the best of available options, then you are really on the same side, you’re just disagreeing about where to draw the line.

      Then you can quit fussing and go have ice cream.

    • Machina ex Deus says:

      I did the math, and Effective Altruism suggests you kill her in an untraceable way, or possibly with whooping cough.

      Sorry, man, but math doesn’t lie.

      @keranih: I have wanted ice cream ever since the St. Basil’s Cathedral thread. Soft-serve, in particular.

      • robirahman says:

        Interesting. Not that I doubt you, but what are your assumptions there? It’s possible if you killed her they’d all just find some different anti-vaxxer crazy lady to follow on facebook instead – I don’t think they’d just suddenly realize the error of their ways and then start vaccinating their kids, thus saving 1000 lives by poisoning the lady.

      • Reasoner says:

        This will make people following her page think there’s an even bigger vaccine coverup.

        Also, thanks for sullying the brand of Effective Altruism by making that comment which associates it with naive interpretations of utilitarianism. You just killed 10^8 future people on expectation. Have a nice day 😛

  25. bean says:

    In the comments to my last post (series index), on pre-dreadnoughts, the difference between turrets and barbettes with gunhouses was thrashed out at some length. I didn’t have a satisfactory explanation at the time, but I’ve spent the last few days groping towards one.
    To recap, the traditional explanation was that turrets are heavy and must thus be mounted low down, meaning low freeboard and poor ability to fight the guns. Barbettes, armored tubes which originally had guns exposed on top which semi-retracted for loading, were lighter and allowed guns to be carried higher in the ship. The barbette mount proved superior, but the exposed guns were an obvious weakness, soon fixed by placing an armored gunhouse over them. However, modern ‘turrets’ are supposedly not turrets in the technical sense because of this, and are somehow lighter than the old turrets.
    The problem comes when you start trying to figure out what the actual difference is. Obviously, there are implementation differences between the turrets on HMS Monarch and the ones on USS Iowa. But despite my best efforts, I can’t find a fundamental disconnect that would break the direct evolutionary lineage.
    The Royal Navy was kind enough to give us a direct comparison of the effects of turrets and unarmored barbettes, in the form of HMS Hood and the Royal Sovereigns respectively. Looking at the armor diagrams in Oscar Parkes’s British Battleships, they both carry the same amount of armor to the same height. The only difference is where the guns are mounted relative to the armor. The weight figures given in Burt, British Battleships 1889-1904, bear this out. There’s not a huge weight savings from the change, although the figures aren’t broken down quite the same, and it’s possible that a hundred tons or so difference exists.
    There is other evidence for this. Norman Friedman, in his US Battleships, does not mention a change in mounting type from the earliest US battleships, which were low-freeboard ships which definitely had turrets, to the later ones. In The Big Gun, Hodges says that the US shifted to barbette/gunhouse mountings in the mid-1890s. A check of Friedman’s Naval Weapons of WWI, which has more details on individual mountings, does not show any radical changes in machinery which we could say correspond to this shift.
    Hodges also says that the French were the only ones to stick to the turret through the end of the pre-dreadnought, along with the Russians, who they greatly influenced. He gives more details, stating that the French kept much of the operating machinery inside the rotating structure, and pushed the guns very high within the turret. The latter is obviously true from photographs, but the former is hard to confirm. I don’t have good drawings of French pre-dreadnought turrets, but the rather poor ones in Naval Weapons of WWI do not show any obvious machinery in the turret. There is a better drawing of a Russian turret, which shows several pieces of machinery outside of the turret.
    In practice, the best way to determine if the reference books will declare something to be a ‘turret’ or a ‘gunhouse’ is to see if it is round. However the turrets in question are sitting on what are clearly barbettes in the modern sense, and are mounted as high as contemporary gunhouses. It seems very odd to draw this distinction, particularly when the US at least went directly from round turrets to turrets that fit more tightly around the guns. (As for why turrets were round when this presumably causes a lot of wasted space, all-angle loading did not become available until the 1890s. Prior to that, the guns had to be pointed in a specific direction, usually fore and aft, to load, which meant that the side of the turret needed to be as heavily armored as the front. More tightly-fitted gunhouses came at the same time as all-angle loading.) Friedman in US Naval Weapons gives another explanation, claiming that turrets are entirely inside the barbette footprint, while gunhouses protrude over the barbette. However, this metric does not match the traditional classification. The Majestics have turrets, while later French and Russian ships are gunhouse/barbette.
    The obvious question is how, if my theory is correct, the return of the turret did not force the guns themselves back down, reducing freeboard. This was because of improvements in armor immediately after the Royal Sovereigns were built. The Royal Sovereigns and Hood had barbette/turret armor of 17”-18”, while the barbettes of the follow-on Majestic class were 14”, with a turret with a 10” face, 5.5” sides and a 4” rear. Thinner armor, combined with a ship that was about 15% larger, meant that the extra topweight from the new gunhouses did not pose a stability problem.
    But if this is the case, why do so many reference books draw the distinction between turrets and barbettes? I believe it has to do with the nature of the debate in Britain. There were many who believed that low freeboard was an important characteristic of a fighting ship, as it made the ship a smaller target. This low freeboard became associated with the turret, as faction which wanted high freeboard was forced into the use of barbette mountings. In the 1880s, these factions seemed to be balanced, as evidenced by the fact that the British went back and forth between turrets and barbettes. In the 1890s, the high-freeboard faction gained the upper hand, but it is possible that the residual power of the low-freeboard group forced them to avoid using ‘turret’ to describe the in-service mountings.

    • Eric Rall says:

      Your posts from the last couple Open Threads has gotten me thinking about to what extent the changes from the Dreadnought Revolution (and similar changes from previous rounds of revised battleship design philosophy) represented recognizing and fixing mistakes in previous designs, and to what extent the previous designs were the best that could be done with the tech at the time while the new ships were taking advantage of technological improvements that made different designs practical and desirable.

      We’ve discussed aspects of this in previous threads, I know. In particular, I remember it being said that mixed main batteries (or intermediate batteries) were a necessary compromise because shortcomings in fire control and rangefinding technology at the time limited the usefulness of the range advantage of big guns, while the rate of fire of the medium guns was so much better that they were worth keeping around. I also seem to remember some discussions around turbine engines (they were a relatively new and unproven tech that in hindsight was clearly better, at least until forced lubrication closed the reliability gap, but they didn’t have a proven track record and nobody before Fisher wanted to risk building a turbine-based battleship that could turn out to be unusable). And those were the two big features that distinguished Dreadnoughts from their immediate predecessors, so Dreadnought-style designs probably weren’t viable more than a few years at most before the started being built in real life.

      Related question which is almost a re-framing of this one: if you fell through a wormhole back to, say, the 1880s (or 1870s, or 1890s), and ship designers from one of the major naval powers somehow realized you were from the future and knew about the future of warship design, and they asked for your advice on their next ship class and you were inclined to give it to them, what would you advise them to build? Something like the best historical designs of that time period, or something ahistorical that incorporated features from future warships?

      • bean says:

        what extent the previous designs were the best that could be done with the tech at the time while the new ships were taking advantage of technological improvements that made different designs practical and desirable.

        Depends on the year. After 1890 or so, the answer is that the designs were coming about as fast as they could. It takes time for changes to percolate through the design process. Usually, they couldn’t have done what they did more than a couple of years earlier. The talk of all-big-gun armament began in 1900 or so, and was realized within 6 years. (A 1902 proceedings article drew early examples of uniform-caliber 12″ and steam turbines.)
        Before 1890, it was a mess everywhere.

        Related question which is almost a re-framing of this one: if you fell through a wormhole back to, say, the 1880s (or 1870s, or 1890s), and ship designers from one of the major naval powers somehow realized you were from the future and knew about the future of warship design, and they asked for your advice on their next ship class and you were inclined to give it to them, what would you advise them to build? Something like the best historical designs of that time period, or something ahistorical that incorporated features from future warships?

        A most excellent question.
        Before the Royal Sovereigns, I’d tell them knock off the turrets for now, except for splinter hoods, and get sufficient freeboards on their ships. And to stop trying to squeeze false economies out of small ships. Try to get the guns right, too.
        After that, I don’t have a whole lot to add, except that they might want to start developing something more than a repeat Majestic a few years earlier. And to pay more attention to fire control, particularly when Pollen shows up. William White was very good, even if I don’t share D.K. Brown’s crush on him.
        For the USN, tell them that the superimposed turret won’t work well (as an aside, I’m amazed that nobody commented on that last time). Other than that, they did a pretty decent job, when you account for US industrial limitations.

  26. Wrong Species says:

    One of my pet peeves is the word “pretentious”. People just deploy it any time someone doesn’t like something popular. It’s actually quite clever. If you just told someone that they were a weirdo for not liking something then you look like a jerk picking on someone for their interests. By using the magic word “pretentious”, you get to pick on someone by insinuating that they think they are better than you are, even if they said nothing of the sort. I get called pretentious all the time no matter how I word my opinions. Anyone else have the same issue?

    • Longtimelurker says:

      I do, but then again I am a bit of a contrarian.

    • HeelBearCub says:

      I get called pretentious all the time no matter how I word my opinions.

      I don’t remember your posts being classifiable as pretentious.

      However, if you are being called pretentious frequently, by all sorts of people, especially people who are not connected to each other, that’s a pretty strong hint that it’s connected to you and your actions.

      Are you being called pretentious mostly in real life settings? What kinds of settings? And why did you specifically mention “not liking something”? Why did you only mention how you word things?

      • Wrong Species says:

        I exaggerated how many times I get called pretentious. It’s usually just my friends. But honestly, what’s the difference between being pretentious and having non-mainstream interests? Isn’t it just how you express your opinions?

        • lvlln says:

          It’s definitely a hazy line, but I don’t think having non-mainstream interests is ever enough to be labeled pretentious. It’s a combination of that and something else, a certain enthusiasm or pride for that interest that exceeds what seems warranted by the audience. For instance, an obsessive anime fan who collects body pillows and figures probably won’t be called pretentious for that, unless he also insists that “it’s ah-nee-may, not ay-nee-may, and never cartoons,” or goes on about how some character’s skin-to-thigh-high ratio makes her absolute territory an S-rank versus some other character’s mere A-rank. These are distinctions that most others simply don’t see as important, and while appreciating them isn’t bad, behaving as if it’s expected that other people also appreciate those differences is where it goes to being “pretentious.”

          In my life, I have a couple of friends who really enjoy fine alcohol and food. These are things I don’t find particularly interesting, but not particularly offensive either. One of my friends is happy to share his experiences and expertise whenever asked, but won’t just bring them up out of the blue or contradict you if you disagree with his tastes. The other is always quick to mention the latest fine wine or bourbon he tried or the fancy meal he had at a 3-month-wait restaurant, and he’s quick to argue or pity if you don’t agree that his experience sounds like the most amazing thing ever. I see the former friend as not pretentious, the latter as pretentious.

          • FacelessCraven says:

            @lvlln – “or goes on about how some character’s skin-to-thigh-high ratio makes her absolute territory an S-rank versus some other character’s mere A-rank.”

            …Is this example drawn from personal encounters? Inquiring weebs wish to know.

        • Brad says:

          Fraiser is quite old at this point, and didn’t age well in many ways, but if you want to see what pretentious looks like, and why it annoys people, watch almost any episode.

        • HeelBearCub says:

          what’s the difference between being pretentious and having non-mainstream interests?

          So, not to play the dictionary game, but, I think the definition is useful:

          attempting to impress by affecting greater importance, talent, culture, etc., than is actually possessed.

          It’s not the having of non-mainstream interests or opinions that is pretentious, it’s the status one attempts to claim by having those opinions.

          In particular, it is the (explicit or implicit) denigration of the interests an preferences of the mainstream that is pretentious.

          For example: Someone says they like “The Big Bang Theory” and you are asked if you like it:
          – Non-Pretentious: “Ehh. It’s not really my kind of show.”
          – Pretentious: “No, I don’t like it and I don’t understand why anyone does.”
          – Super-Pretentious: “Oh heaven’s that show is so derivative. The first season was passable entertainment, but how anyone could be watching it now is beyond me.”

          • Thegnskald says:

            I understand why normal people like Big Bang Theory.

            I cannot understand why nerds ever enjoy it; it is basically nerd blackface.

          • The Nybbler says:

            I cannot understand why nerds ever enjoy it; it is basically nerd blackface.

            The objection to blackface basically _only_ applies to blackface. It can’t be generalized to “non-member of group plays stereotypical character of group”.

          • Thegnskald says:

            Do you say this descriptively, or prescriptively?

            Certainly I do not think it is acceptable to mock people in this fashion. “Jocks” get no better a treatment, and I think a not insignificant amount of animosity between the two groups comes from their portrayals more than anything latent.

          • Matt M says:

            I think Big Bang Theory, while inaccurate and exaggerated in many ways, basically has its “heart in the right place.” Pretty much everyone except Sheldon is portrayed as a relatively nice, normal, and high-value person who just happens to be into nerdy things.

            And even Sheldon is portrayed much more positively and gets much better results than anyone who actually behaved like him would ever hope to receive in real life. ALL of the characters are portrayed as people who are worthy of love, respect, and friendship, and not JUST from other nerds, but from normal people like Penny as well. I don’t really see what there is to be outraged about (and I say that as someone who gets compared to Sheldon in real life frequently)

          • rlms says:

            I agree with Matt M: it’s not funny but it isn’t offensive.

          • Jiro says:

            The objection to blackface basically _only_ applies to blackface. It can’t be generalized to “non-member of group plays stereotypical character of group”.

            I can think of objections to blackface that also apply to other groups. For instance, it’s more likely that the non-member won’t know how to use the stereotype respectfully, and it’s more likely he won’t understand that some stereotypes are inaccurate or even are stereotypes at all.

            (Writers are probably more important here, but actors impart enough of a spin through their acting that they have some effect as well.)

            Also, non-members portraying members may have bad historical connotations. Obviously this isn’t as bad for nerds as for blacks, but nerds were treated more harshly within living memory.

          • rlms says:

            lol

          • John Schilling says:

            Obviously this isn’t as bad for nerds as for blacks, but nerds were treated more harshly within living memory.

            Within living memory of people under fifty who live entirely in middle-class areas of cities and suburbs, maybe.

          • Incurian says:

            Hey, don’t deny his lived experience, etc.

          • LHN says:

            FWIW, I read “more harshly” as “more harshly than nerds are now” rather than “more harshly than African-Americans”.

            (But also saw the unfortunate ambiguity.)

      • Matt M says:

        Disliking a popular thing is necessary, but not sufficient, to be pretentious. You also have to have a certain dismissive attitude of things that are mainstream/popular.

        If someone asks you your opinion on mainstream/popular thing and you say “I don’t really care for that thing” that’s generally fine and not pretentious. If you walk around loudly broadcasting your unsolicited opinion that popular things suck, that’s pretentious – or if you respond to a question with “I don’t even understand how anyone could like this popular thing” or something similarly dismissive, that’s pretentious as well.

        I’d also suggest that if you dislike popular things in ALL categories, across the board, that will definitely come across as pretentious, because what are the odds of you holding those opinions if not for some ulterior motive (such as hating popular things solely due to their popularity in and of itself)

    • Anonymous says:

      Maybe your opinions are pretentious? Can you give some examples of what comments triggered accusations of pretentiousness?

    • rlms says:

      I think accusation of pretentiousness usually come from belittling other people’s opinions, rather than having weird ones yourself.

    • Fahundo says:

      It’s just a form of bulverism. People use contrarian in pretty much the same way now. It’s an easy way to dismiss an outlying point of view without having to engage with it.

  27. dndnrsn says:

    This is the kind of place where people would know this sort of thing (I’m especially thinking @bean might know something of it): are there any good resources for figuring out the details of travel (especially sea travel) in the past – specifically, things like price, time, and layovers.

    I’m running a roleplaying game set in the early 20th century, and the PCs’ travel arrangements are important enough to require details. I’ve been doing some searching, but it all seems to be present-day stuff for the nitty-gritty; stuff relating to the past tends to be far more articles about what it was like to travel on a liner or in a DC-3 back in the day. Dealing with travel by train is not a huge problem – I just look up modern day routes and assume everything would take a bit longer and take a stab at guessing prices based on current-day prices – I do not need a great level of accuracy, but rather the ability to say “it costs you x dollars and takes y days to get from here to there” without it being complete nonsense) but dealing with intercontinental travel is a bother. The capabilities of planes are significantly better than in the olden days, and as a result travel by sea seems to mostly have become a luxury cruise thing; it seems rather difficult to find good concrete details as to oceanic travel for people (as opposed to cargo shipping today) and air travel (very few flights nowadays feature layovers, whereas planes with very limited ranges required a lot of layovers).

    If anyone knows a decent resource or way to figure this out, please let me know. Thanks.

    • massivefocusedinaction says:

      This paper has some ship fare trends.
      http://eh.net/eha/wp-content/uploads/2013/11/Weissetal.pdf

      The Lusitania’s wiki page lists its record setting Atlantic crossing times, so anything in the 6 to 9 day range would seem reasonable for an Atlantic Crossing (you could probably look for a few modern re-positioning cruise or Cunard schedules and be close for powered ship crossings). Powered passenger ships aren’t designed for sustained speed, because people mostly cross the Atlantic for fun, these days, so being faster isn’t a competitive advantage.

      I would think there would be some data preserved with the US regulated air fares to various locations after 1938, but I don’t know where to look for it.

    • Nornagest says:

      Well, there’s one very famous example that shouldn’t be too hard to mine for data: third-class tickets on the Titanic cost between $15 and $40 in 1912 dollars, or roughly $175-$450 in today’s money, and prices went up from there in roughly the proportions we’re used to for airline business- or first-class service. The trip was expected to take six or seven days.

      I don’t know anything about air travel in those days. Intercontinental flight would not have been practical, unless you’re in the interwar period and talking about zeppelins, but you knew that.

      • dndnrsn says:

        Are you sure you’ve got the inflation etc right? Googling suggests $40 in 1912 dollars is about a grand today.

        • Nornagest says:

          No, I’m not. That was a close paraphrase from the source I Googled, which might be old or wrong. bls.gov says you’re right, although adjusting for inflation over that kind of timescale is a little bit fraught.

      • Skivverus says:

        Considering that Lindbergh’s flight was in the mid-1930s 1920s, “would not have been practical” is probably a bit of an understatement.

        (You’d think I’d remember the date for ten minutes after looking it up on Wikipedia, but apparently not)

        • Nornagest says:

          1927, actually. I didn’t know what “early 20th century” meant specifically, and depending on the period a nonstop flight would have been anything from totally impossible, to possible with a few stock aircraft but little useful cargo, so “impractical” seemed to cover the bases. Very late in the ’30s, it was possible to book an intercontinental flight, but you’d have been using flying boats and paying a ton of money, and I only know of one that could have made the flight without refueling mid-route.

        • LHN says:

          Well, 1927. And his was the first solo flight; the first transatlantic flight was eight years earlier. But commercial transatlantic travel didn’t start till 1939, and was promptly interrupted by WWII. Before that, if the characters really want to fly across the sea, they’ll need money and some reasonably daring charter pilots.

    • LHN says:

      Rail has such an obsessive fandom (and I mean that with admiration) that I’m sure you can do better for train schedules and travel times than extrapolating from current ones.

      A quick Google turns up “Streamliner Schedules”, which looks like it aims to cover the US during the period you’re looking into. http://www.streamlinerschedules.com/ and Timetable World which has a broader ambit: http://timetableworld.com/ I’m morally certain there’s quite a bit more.

      • At 6:00 am, on March 6, 1930, a congressman suffered a heart attack and died instantly. Standard sources say he died “near Washington, DC”, but I found that unsatisfactory.

        He died on a Chesapeake & Ohio train en route from Huntington, WV to Washington DC, which was regularly scheduled and arrived on time at 7:40 am. A railfan organization in Philadelphia provided photocopies copies of schedules which helped me to determine approximately where the train was when it happened.

        See http://politicalgraveyard.com/bio/glovinsky-gochenour.html#384.61.91

    • LHN says:

      For ships, this looks like it might be helpful:

      Passenger Fares for Overseas Travel in the 19th and 20th Centuries: http://eh.net/eha/wp-content/uploads/2013/11/Weissetal.pdf

      At the end, there’s a handy graph of trends in first, second, and third class ticket prices.

      (Whoops– I’d missed that massivefocusedinaction had already posted this one.)

      For a picture of the overall experience, I’d recommend The Only Way to Cross by John Maxtone-Graham. (Not yet available as an ebook, alas, but in a lot of libraries, and cheap used via Amazon.)

    • LHN says:

      For air, Airline Timetable Images, http://www.timetableimages.com/index.htm appears to have a fairly huge collection of scanned historical timetables.

      Though the ones I looked at were a little clunky, since each page had to be clicked through as a single JPEG. E.g., here’s a page from Central Airlines (“2 Pilots On Every Airliner”!) in 1936, showing fares and schedules: http://www.timetableimages.com/ttimages/cen/cen3607/cen3607i.jpg

    • bean says:

      Unfortunately, I don’t have any particular sources on transportation that far back. The only thing that springs to mind is to look in the 1911 Brittanica, which is out of copyright and has a downright astonishing level of detail on a bunch of things. I plan to set any historical game I run ~100 years ago in 1911 to take advantage of it.
      Given that you’re talking about air travel, though, I don’t think this is likely to be in the right era. I’ve only had to go looking for deep historical data a few times, and it’s usually for comparison to modern data, so I take what I can get.

      • LHN says:

        The 1911 Britannica is awesome. I’ve long had my eye on my dad’s copy (acquired for a song at a long ago library book sale), even though our bookshelves don’t remotely have the space for it.

    • nimim.k.m. says:

      For some reason my attempt to link a Cthulhu RPG forum where exactly this kind of stuff is (or was when I was active) discussed is getting stuck in some ban filter. (First I posted things that for some reason did not get through the filter, removed random stuff in the hopes it could get through the filter, and woah, finally it got! Then I edited the url to my message and the message disappeared, sigh.) Try to google by yourself.

      One recommendation I got from there years ago: out-of-copyright Baedeker travel guides from the early 1900s are available on the Internet Archive, and contain exactly this kind of information. For example, (from the 1906 guide to the United States), travel across Atlantic from Europe to US took 6-9 days, and cost $75-125. Faster steamers were more expensive than slow ones.

    • MrApophenia says:

      The Call of Cthulhu published campaign, Masks of Nyarlathotep, is a globe-trotting campaign set in the 1920s, and there is also a ticking clock element of the story, so there’s a whole section on how to manage exactly this, including shorthand summaries of how long it takes to get between various continents by various means. It should be about exactly what you are looking for.

      Plus, it is maybe the best published RPG campaign, certainly top ten, so it’s worth getting for the rest of it as well.

      • dndnrsn says:

        I am no stranger to Masks. That every globetrotting campaign doesn’t include a chart like that, or information beyond vague “it’s cheaper and faster to travel across the Atlantic than the Pacific” is actually the exact problem I am having. The chart in Masks is for location to location, so for a campaign with different locations, it isn’t as useful.

        It’s definitely the best campaign I’ve ever played. It alone makes the game worth it.

  28. Winter Shaker says:

    This is maybe a bit forward (both in presumptuousness and in time-into-the-future), but in the spirit of SSC meetups everywhere, I am looking likely to be in the Netherlands for a folky dance festival in October this year, and if Aapje and any other Dutch or Dutch-adjacent people are on the way (the venue is a bit east of Arnhem), I’d be up for a get-together. Ik hoof dat ik op dat tijd meer nederlands zal kunnen spreken.

    • Aapje says:

      Cadansa, I guess?

      Arnhem is relatively far away (although Americans seem to define ‘far away’ as larger distances). If you land at Schiphol, it may however be fairly easy for us to meet, if you land or leave during/near business hours. You can mail me at [wouter.spam at gmail.com] (just a redirect address that I use for risky signups or in this case, to keep my identity secret).

      PS. typo/mistake in your Dutch: hoof should be hoop.

      • Winter Shaker says:

        That’s the one – good to hear it’s already known to you. I’m about the last member of my clique of folky dancers to have not been yet. Anyway, I’ll see if that’s a sensible route to travel, and if so, give you a buzz.

  29. Nabil ad Dajjal says:

    I was hoping one of you guys with a physics background could help me answer a question about Alcubierre drives. It’s not terribly important but it’s interesting to think about and has bearing on a semi-hard SF idea I’ve been developing.

    A 2012 paper on the “warp drive” concept concludes:

    These results suggest that any ship using an Alcubierre warp drive carrying people would need shielding to protect them from potential dangerously blueshifted particles during the journey, and any people at the destination would be gamma ray and high energy particle blasted into oblivion due to the extreme blueshifts for P+ region particles.

    The issue about the ship needing shielding sounds a lot like the issue mentioned in Semiclassical instability of dynamical warp drives where Hawking radiation would fry the ship. The interesting bit to me is this quote from their conclusion:

    In conclusion, we think that this work is casting strong doubts about the semiclassical stability of superluminal warp drives. Of course, all the aforementioned problems disappear when the bubble remains subluminal. In that case no horizons form, no Hawking radiation is created, and neither strong temperature nor white horizon instability is found. The only remaining problem is that one would still need the presence of some amount of exotic matter to maintain the subluminal drive.

    So my question is, would a ship with an Alcubierre drive travelling just under the speed of light fry everything in front of it with blueshifted particles? Or is that another issue which only applies to superluminal travel? What about travel at exactly light speed?

    • MNH says:

      I’ve only finished physics undergrad, and I learned enough to mostly understand the first article, but not at all the second. I’ll take a stab anyways, but take this with a grain of salt.

      The two things that you’re reading are quite different. The first doesn’t give a shit about quantum effects, and is just describing a mechanism by which a bunch of particles would accumulate (kind of like a sonic boom). The paper says “When the bubble velocity is subluminal, all forward
      and backward moving light rays pass through the bubble” (note if you read this section that “gamma rays” and “null particles” are essentially other ways of saying “light”), so light will definitely not contribute to this effect at subluminal speeds. The paper also states that for massive particles (ones that have mass at all, they don’t have to be ‘massive’ in the colloquial sense), there’s a critical particle velocity (v_{c}) for each subluminal speed, and massive particles moving above that velocity will become trapped (and therefore should, I think, contribute to this sonic boom effect). I guess this all means that if you’re subluminal, whether it’s dangerous or not depends on how many massive particles moving forward with you at or above v_{c} you’ll encounter on your trip. Massive particles will always be captured if you are going at luminal or subluminal velocities, unless the particles have no velocity themselves, in which case the paper says it confirms a result of some dudes (or chicks) Pfenning and Ford, who found that unmoving massive particles will never be captured. I cannot find any info on whether an exactly-luminal ship will capture light. Also, note that all massive particle velocities are measured in the reference frame of the origin/destination.

      I actually feel pretty confident up to here. The second article is where this really gets over my head. Semiclassical here means that the gravity is at least normal (we don’t know how quantum gravity works), but all the matter is quantum and doing weird field shit that I’ve never learned. If it says all its problems disappear when you’re subluminal, trust it, I guess.

      • Nabil ad Dajjal says:

        Thanks!

        I really appreciate the response, it’s challenging for me to understand this stuff. Without your help I wouldn’t have been able to figure that out.

        • MNH says:

          Sure, although proofreading I just noticed an error.
          Where I said:

          Massive particles will always be captured if you are going at luminal or subluminal velocities, unless…

          It should read:

          Massive particles will always be captured if you are going at luminal or superluminal velocities, unless…

  30. SolveIt says:

    Is taking an IQ test (say, the Wechsler) worth the money? What new actionable information can I expect given that I already have a vague idea of how smart I am (gut feeling, standardized exams, etc)?

    • Scott Alexander says:

      Probably not. I agree it wouldn’t provide you with actionable information.

    • Jaskologist says:

      Which is worse?

      1. Finding out that you’re not as smart as you think.
      2. Finding out that you are as smart as you think.

    • Glen Raphael says:

      These kind of tests mostly tell you what you already know. If you just enjoy taking brain tests and getting a reassuring numeric score, there are cheaper ways to do that. Me, I enjoy and recommend a brain-training smartphone app called Peak.

      What new actionable information can I expect

      From WAIS I learned I am relatively bad at “digit span” compared to other mental skill categories – it’s not a strength. From Peak, I learned I am relatively bad at “emotion” (primarily recognizing facial expressions) compared to everything else. I kind of already knew these things – they weren’t actionable info for me, but YMMV.

      If you try Peak, you might be surprised at how LOW your scores start out, until you realize they are norming results not relative to people your age in the general population, but relative to “active Peak users” – people your age who can afford a smartphone and enjoy taking brain tests enough to buy an app which nags them to do that regularly. (Also, the tests are trainable, so the numbers quickly improve.)

    • Anonymous says:

      IMO, yes. Mensa offers them on the cheap where I am.